SlideShare una empresa de Scribd logo
1 de 12
Descargar para leer sin conexión
MÓDULO I – PARTE 6                          MATEMÁTICA
                 Projeto
                Vestibular                    Noções de                                  Prof. Bruno Vianna
                                            Probabilidade

             Noções de Probabilidade                               Dividindo todos os membros da desigualdade
                                                           por n(U), vem:

       Experimento determinístico                                        0    n( A ) n(U)
                                                                            ≤       ≤          ∴ 0 ≤ P( A ) ≤ 1
                                                                        n(U) n(U) n(U)
        Experimentos que ao serem realizados
repetidas vezes em condições consideradas idênticas,
apresentam resultados essencialmente idênticos são                   Probabilidade de Não Ocorrer Um Evento
denominados experimentos determinísticos.
                                                                  Sendo A evento complementar do evento A do
                                                           espaço amostral U, temos:
       Experimento aleatório

        Experimentos que ao serem realizados                                       P( A ) + P( A ) = 1
repetidas vezes em condições consideradas idênticas,
apresentam resultados diferentes, não sendo possível
portanto a previsão lógica dos resultados, são             Exercícios Resolvidos
denominados experimentos aleatórios (ou casuais).
                                                           01) Uma urna contém 15 bolas numeradas de 1 a 15.
                                                           Uma bola é extraída ao acaso da urna. Qual a
       Espaço Amostral                                     probabilidade de ser sorteada uma bola com número
                                                           maior ou igual a 11?
       É o conjunto de todos os resultados possíveis
de um experimento aleatório. Indicaremos o espaço          Espaço amostral U = {1,2,3,...,13,14,15}
amostral por U.
                                                           Evento requerido A = {11,12,13,14,15}
                                                           (Nºs maiores ou iguais a 11)
       Evento
                                                           n(A) = 5
        É qualquer subconjunto do espaço amostral.         n(U) = 15
        O conjunto Ø é chamado evento impossível.
        O conjunto espaço amostral U é também um                     n( A) 5 1
evento, chamado de evento certo.                           p( A) =        =  = ≅ 33,3%
        Os subconjuntos unitários de U são chamados                  n(U ) 15 3
eventos elementares ou eventos simples.
                                                           É certo que também podemos simplificar a idéia de
                                                           probabilidade quando as situações estudadas são
       Espaço Amostral Eqüiprovável                        de fácil compreensão:
                                                                n º de casos favoráveis 5 1
       O espaço amostral de um experimento aleatório       p=                           =   = ≅ 33,3%
é chamado eqüiprovável se todos os seus eventos
                                                                     n º total de casos   15 3
elementares têm a mesma chance de ocorrer

                                                           02) Um dado é lançado e observa-se o número da face
       Probabilidade                                       voltada para cima. Qual a probabilidade desse número
                                                           ser:
       Seja U um espaço amostral eqüiprovável e A                                2 1
um de seus eventos. Denomina-se probabilidade do           a) menor que 3?    p=   = ≅ 33,3%
evento A o número P(A) tal que:                                                  6 3
                               n( A )                                                4 2
                    P( A ) =                               b) maior ou igual a 3? p = = ≅ 66,6%
                               n(U)                                                  6 3
       onde:
       n(A) = número de elementos do evento A.                                              1 2
       n(U) = número de elementos do espaço                Observe que P(A) + P(B) =         + =1
amostral U.                                                                                 3 3
                                                           Ou seja, como P( A ) + P( A ) = 1 , temos que   P( B) = P( A)
       Como A é subconjunto de U, decorre que:
                  0 ≤ n(A) ≤ n(U)
                                                                                                                    2011
                                                       1
MÓDULO I – PARTE 6                       MATEMÁTICA
                    Projeto
                   Vestibular                           Noções de                               Prof. Bruno Vianna
                                                      Probabilidade

           Probabilidade da união de eventos                                   12 8   4 16
                                                               p( A ∪ B) =       +  −  =   = 0,64 = 64%
                                                                               25 25 25 25
       Se A e B são eventos quaisquer de um
experimento aleatório do mesmo espaço amostral U,              b) Qual a probabilidade do nº da bola sorteada ser
então:                                                         múltiplo de 5 ou de 7?

              n( A U B) = n( A ) + n(B) − n( A I B)                                       5
                                                               Múltiplos de 5 >    p ( A) =
                                                                                          25
       Dividindo ambos os membros dessa igualdade                                          3
por n(U), temos:                                               Múltiplos de 7 > p ( B ) =
                                                                                          25
             n( A U B) n( A ) n(B) n( A I B)
                      =      +    −
               n(U)     n(U) n(U)    n(U)                      Múltiplos de 5 e 7 >      p( A ∩ B) = Ø

        Onde concluímos que:                                                   5   3   8
                                                               p( A ∪ B) =       +   =   = 0,32 = 32%
             P( A U B) = P( A ) + P(B) − P( A I B)                             25 25 25

         Pode ocorrer que os eventos A e B do                                      Probabilidade Condicional
espaço amostral U não tenham elementos comuns.
Nesse caso, são chamados de eventos mutuamente                          Denomina-se      probabilidade de          B
exclusivos ( ou eventos disjuntos ). Quando isso               condicionada a A a probabilidade de ocorrência do
ocorre, temos:                                                 evento B, sabendo que vai ocorrer ou que já ocorreu o
                                                               evento A. Representaremos esse caso por P( B | A )
              A IB = { }      ⇒      P(A I B) = 0              (lê-se probabilidade de B dado A ).

        Logo, se A e B são eventos mutuamente
exclusivos, temos:                                                     U                                         B
                                                                               A
                   P( A U B) = P( A ) + P(B)



                        Resumindo:

       p( A ∪ B ) = p ( A) + p( B) ⇔ A ∩ B = Ø                                                A∩
                              ou                                                              B
 p( A ∪ B ) = p ( A) + p( B) − p( A ∩ B ) ⇔ A ∩ B ≠ Ø                 Observe que, sabendo que o evento A ocorreu,
                                                               então os casos favoráveis à ocorrência do evento B
Exercício Resolvido:                                           estão em A ∩ B.
                                                                      Temos então:
03) Uma urna contém 25 bolas numeradas de 1 a 25.
                                                                                                     n( A I B)
Uma bola é extraída ao acaso.                                                          P(B | A ) =
                                                                                                       n( A )
a) Qual a probabilidade de o nº da bola sorteada ser
múltiplo de 2 ou de 3?                                               Dividindo numerador e denominador do
                                                               segundo membro da igualdade por n(U), temos:
                         12
Múltiplos de 2 >   p ( A) =                                                       n( A I B)
                         25                                                         n(U)                              P( A I B )
                                                                      P(B | A ) =               ==> P(B | A ) =
                         8                                                          n( A )                              P( A )
Múltiplos de 3> p( B ) =
                         25                                                         n(U)


                                        4                              Logo:
Múltiplos de 2 e 3 >   p( A ∩ B) =
                                        25                                          P( A I B ) = P( A ) ⋅ P(B | A )

                                                                                                                                   2011
                                                           2
MÓDULO I – PARTE 6                     MATEMÁTICA
                    Projeto
                   Vestibular                          Noções de                          Prof. Bruno Vianna
                                                     Probabilidade

        Então, para a ocorrência ao mesmo tempo de              02) (UNI-RIO) – O dispositivo que aciona a abertura do
dois eventos, temos que a probabilidade de ocorrer A e          cofre de uma joalheria apresenta um teclado com nove
B é igual à probabilidade de ocorrer A multiplicada pela        teclas, sendo cinco algarismos (0,1,2,3,4) e quatro letras
probabilidade condicional de B dado A.                          (x,y,z,w). O segredo do cofre é uma seqüência de três
                                                                algarismos seguidos de duas letras. Qual a
        Os eventos A e B são chamados eventos
                                                                probabilidade de uma pessoa, numa única tentativa, ao
independentes ou seja, a ocorrência de um evento não
                                                                acaso, abrir o cofre ?
depende da ocorrência do outro, quando vale a
igualdade:
                                                                (A) 1 / 7 200           (B) 1 / 1 500
                   P( A I B) = P( A ) ⋅ P(B)                    (C) 1 / 2 000           (D) 1 / 720
                                                                (E) 1 / 200
Exercícios Resolvidos
                                                                03) (UNIRIO-2000) Numa urna existem bolas de
04) Uma urna contém exatamente sete bolas: quatro               plástico, todas do mesmo tamanho e peso, numeradas
azuis e três vermelhas. Retira-se ao acaso uma bola da          de 2 a 21, inclusive e sem repetição. A probabilidade de
urna, registra-se sua cor e repõe-se a bola da urna. A          se sortear um número primo ao pegarmos uma única
seguir, retira-se novamente uma bola da urna e registra-        bola, aleatoriamente, é de:
se sua cor. Calcular a probabilidade de:
                                                                (A) 45%         (B) 40%         (C) 35%
a) sair uma bola azul e outra vermelha.                         (D) 30%         (E) 25%

                                                                04) (UERJ-02) Em uma experiência de fecundação in
                                                                vitro, 4 óvulos humanos, quando incubados com 4
                                                                suspensões de espermatozóides, todos igualmente
Queremos que a primeira bola retirada seja azul e a             viáveis, geraram 4 embriões, de acordo com a tabela
segunda seja vermelha. A probabilidade de a primeira            abaixo.
bola ser azul é 4 , e a probabilidade de a segunda bola
                7
sair vermelha é 3 . Assim, a probabilidade de obtermos
                 7
a sequência: A e V é P = 4 ⋅ 3 = 12
                            7 7 49

b) saírem duas bolas de cores diferentes.                       Observe os gráficos:

Temos duas sequências possíveis, com as respectivas
probabilidades:

A e V → P1 = 4 ⋅ 3 = 12     OU V e A → P2 = 3 ⋅ 4 = 12
             7 7 49                         7 7 49

Assim a probabilidade total é: P = P1 + P2 = 12 + 12 = 24       Considerando a experiência descrita, o gráfico que
                                             49 49 49           indica as probabilidades de os 4 embriões serem do
                                                                sexo masculino é o de número:
Exercícios Propostos
                                                                (A) 1           (B) 2           (C) 3           (D) 4
01) Estudando Genética, os alunos da E.A. Corcovado
construíram o quadro ao lado, em que os quatro                  05) (UERJ-06-2ºex) Com o intuito de separar o lixo para
eventos são prováveis. Qual a probabilidade de que              fins de reciclagem, uma instituição colocou em suas
ocorra o evento aa (em que o filho de um casal híbrido          dependências cinco lixeiras de diferentes cores, de
de olhos castanhos teria olhos azuis) ?                         acordo com o tipo de resíduo a que se destinam: vidro,
Masc  Fem             A                      a                 plástico, metal, papel e lixo orgânico.
      A         AA (castanho)           Aa (castanho)
      A         Aa (castanho)             aa (azul)

(A) 50%          (B) 25%                       (C) 75%
(D) 10%          (E) 20%
                                                                                                                     2011
                                                            3
MÓDULO I – PARTE 6                                 MATEMÁTICA
                  Projeto
                 Vestibular                      Noções de                                    Prof. Bruno Vianna
                                               Probabilidade

Sem olhar para as lixeiras, João joga em uma delas             11) (UERJ)
uma embalagem plástica e, ao mesmo tempo, em outra,
uma garrafa de vidro.
A probabilidade de que ele tenha usado corretamente
pelo menos uma lixeira é igual a:
(A) 25%         (B) 30%        (C) 35%         (D) 40%

06) (OBMEP-05) Brasil e Argentina participam de um
campeonato internacional de futebol no qual competem
oito seleções. Na primeira rodada serão realizadas
quatro partidas, nas quais os adversários são
escolhidos por sorteio. Qual é a probabilidade de Brasil
e Argentina se enfrentarem na primeira rodada?
                                                                  Protéticos e dentistas dizem que a procura por
(A) 1/8         (B) 1/7        (C) 1/6                         dentes postiços não aumentou. Até declinou um
(D) 1/5         (E) 1/4                                        pouquinho. No Brasil, segundo a Associação Brasileira
                                                               de Odontologia (ABO), há 1,4 milhão de pessoas sem
07) (PM-05-1) Pedro brinca com um dado com seus                nenhum dente na boca e 80% delas já usam dentadura.
amigos. Ele não gosta do número 3. Se Pedro lançar o           Assunto encerrado.
dado duas vezes, a probabilidade de que o número 3             (Adaptado de Veja, outubro/97)
não apareça em nenhum dos lançamentos é de,
aproximadamente:                                                  Considere que a população seja de 160 milhões de
                                                               habitantes.
(A) 40%         (B) 50%        (C) 60%         (D) 70%            Escolhendo ao acaso um desses habitantes, a
                                                               probabilidade de que ele não possua nenhum dente na
08) (PM-04-2) Em certo quartel, a probabilidade de um
                                                               boca e use dentadura, de acordo com a ABO, é de:
soldado ser torcedor do Flamengo é 0,60 e de gostar de
praticar exercício de tiro é 0,70. As probabilidades
                                                               (A) 0,28%;       (B) 0,56%;      (C) 0,70%;     (D) 0,80%.
mínima e máxima de um soldado deste quartel ser
torcedor do Flamengo e, simultaneamente, gostar de
praticar exercícios de tiro, são, respectivamente:             12) Numa urna contendo 5 bolas brancas e 10 bolas
                                                               pretas, cada vez que se retira uma delas procede-se da
(A) 10% e 60%                  (B) 20% e 60%                   seguinte maneira:
(C) 30% e 60%                  (D) 40% e 60%                    − Se a bola for branca: não se repõe esta bola,
                                                                 porém acrescenta-se 6 outras bolas pretas;
09) (PM-04-2) Um comandante deseja premiar três dos
                                                                − Se a bola for preta: repõe-se esta bola
sete soldados mais qualificados de seu quartel,
                                                                 juntamente com outras 5 bolas brancas.
adotando o critério de sorteio. Todos os soldados
qualificados têm nomes diferentes e João e Pedro estão            A probabilidade da SEGUNDA bola retirada desta urna ser
entre eles. A probabilidade de João e Pedro serem dois         branca, é:
dos nomes sorteados é de:
                                                               (A) 20%      (B) 25%        (C) 33,333...% (D) 40% (E) 50%
(A) 1/7         (B) 2/7        (C) 3/7         (D) 4/7
                                                               13) Uma urna A contém x bolas vermelhas e y bolas
10) (UERJ-99)
                                                               brancas. Uma urna B contém z bolas vermelhas e w
                                                               bolas brancas. Uma bola é retirada da urna A e
                                                               colocada na urna B e, então, uma bola é retirada da
                                                               urna B. A probabilidade desse última bola ser vermelha
                                                               é:

                                                                       z +1                                x+z
(O Dia, 25/08/98)                                              (A)                                 (B)
                                                                     z + 1+ w                            x+y+z+w
Suponha haver uma probabilidade de 20% para uma
caixa de Microvlar ser falsificada. Em duas caixas, a                  1  x + xz + zy                    1  xy + xz + zy 
                                                               (C)                               (D)
                                                                     x + y  z + w +1 
                                                                                                                           
probabilidade de pelo menos uma delas ser falsa é:                                                     x + y  z + w +1 

(A) 4 %      (B) 16 %       (C) 20 %       (D) 36 %


                                                                                                                          2011
                                                           4
MÓDULO I – PARTE 6                          MATEMÁTICA
                  Projeto
                 Vestibular                      Noções de                                Prof. Bruno Vianna
                                               Probabilidade
14) (Enem-2001) Uma empresa de alimentos imprimiu              16) (PUC-RIO-2010) Quatro moedas são lançadas
em suas embalagens um cartão de apostas do seguinte            simultaneamente. Qual é a probabilidade de ocorrer
tipo:
                                                               coroa em uma só moeda?

                                                                   1               2                 1
                                                               (A)             (B)             (C)
                                                                   8               9                 4
                                                                   1               3
                                                               (D)             (E)
                                                                   3               8

                                                               17) (PUC-RIO-2011) Jogamos três dados comuns
                                                               simultaneamente. Qual a probabilidade de que os três
                                                               números sorteados sejam distintos?
                                                                   1                1                5
                                                               (A)             (B)             (C)
                                                                   2               36                9
                                                                   17               5
                                                               (D)             (E)
                                                                   36              17

                                                               18) (Enem-2001) Um município de 628 km² é atendido
                                                               por duas emissoras de rádio cujas antenas A e B
                                                               alcançam um raio de 10km do município, conforme
                                                               mostra a figura:




                                                               Para orçar um contrato publicitário, uma agência precisa
                                                               avaliar a probabilidade que um morador tem de,
Cada cartão de apostas possui 7 figuras de bolas de            circulando livremente pelo município, encontrar-se na
futebol e 8 sinais de “X” distribuídos entre os 15             área de alcance de pelo menos uma das emissoras.
espaços possíveis, de tal forma que a probabilidade de
um cliente ganhar o prêmio nunca seja igual a zero.
                                                               Essa probabilidade é de, aproximadamente,
Em determinado cartão existem duas bolas na linha 4 e
duas bolas na linha 5. Com esse cartão, a probabilidade        (A) 20%.        (B) 25%.        (C) 30%.
de o cliente ganhar o prêmio é                                 (D) 35%.        (E) 40%.
(A)1/27.       (B) 1/36.       (C) 1/54.                       19) (PUC-RIO-2011) Considere uma urna contendo
(D)1/72.       (E) 1/108.                                      vinte bolas numeradas de 1 a 20. Retiram-se três bolas
                                                               simultaneamente e de maneira aleatória de dentro desta
15) A figura abaixo representa um alvo de dardos,              urna.
composto de três círculos concêntricos de raios r, 2r e
3r. Sabendo que um competidor acertou o alvo, qual é           a) Qual a probabilidade de que a soma seja igual a 6?
a probabilidade dele ter acertado a parte clara do alvo?       b) Qual a probabilidade de que a soma seja igual a 8?
                                                               c) Qual a probabilidade de que a soma seja igual a 15?

                                                               20) (PUC-RIO-2011) Considere uma urna contendo 5
                                                               bolas pretas e 5 bolas brancas. Retiram-se
                                                               simultaneamente e de maneira aleatória 3 bolas de
                                                               dentro desta urna.

                                                               a) Qual a probabilidade de que todas as bolas retiradas
                                                               sejam brancas?
(A) 1/3        (B) 1/2         (C) 1/4                         b) Qual a probabilidade de que, entre as bolas retiradas,
(D) 1/9        (E) 4/9                                         duas bolas sejam brancas e uma bola seja preta?
                                                                                                                   2011
                                                           5
MÓDULO I – PARTE 6                                    MATEMÁTICA
                            Projeto
                           Vestibular              Noções de                                               Prof. Bruno Vianna
                                                 Probabilidade

21) (ENEM-2010) O diretor de um colégio leu numa                 Em 2050, a probabilidade de se escolher, aleatoriamente,
revista que os pés das mulheres estavam aumentando.              uma pessoa com 60 anos ou mais de idade, na população
Há alguns anos, a média do tamanho dos calçados das              dos países desenvolvidos, será um número mais próximo
mulheres era de 35,5 e, hoje, é de 37,0. Embora não              de
fosse uma informação científica, ele ficou curioso e fez             1                          7                           8
uma pesquisa com as funcionárias do seu colégio,                 (A)                       (B)                      (C)
                                                                     2                         20                           25
obtendo o quadro a seguir:
                                                                     1                         3
                                                                 (D)                       (E)
Escolhendo uma funcionária ao acaso e sabendo que                    5                         25
ela tem calçado maior que 36,0, a probabilidade de ela
calçar 38,0 é:                                                   23) (ENEM-09) O controle de qualidade de uma
                                                                 empresa fabricante de telefones celulares aponta que a
                                                                 probabilidade de um aparelho de determinado modelo
                                                                 apresentar defeito de fabricação é de 0,2%. Se uma loja
                                                                 acaba de vender 4 aparelhos desse modelo para um
                                                                 cliente, qual é a probabilidade de esse cliente sair da
                                                                 loja com exatamente dois aparelhos defeituosos?
                                                                                       4                                          2
                                                                 (A) 2 × (0,2%) .                                   (B) 4 × (0,2%) .
                                                                                       2                  2
    1                       1              2                     (C) 6 × (0,2%) × (99,8%) .                         (D) 4 × (0,2%).
(A)                     (B)          (C)
    3                       5              5
                                                                 (E) 6 × (0,2%) × (99,8%).
    5                        5
(D)                     (E)
    7                       14                                   24) (ENEM-09) A população brasileira sabe, pelo menos
                                                                 intuitivamente, que a probabilidade de acertar as seis
22) (ENEM-09) A população mundial está ficando mais              dezenas da mega sena não é zero, mas é quase.
velha, os índices de natalidade diminuíram e a expectativa       Mesmo assim, milhões de pessoas são atraídas por
de vida aumentou. No gráfico seguinte, são apresentados          essa loteria, especialmente quando o prêmio se
dados obtidos por pesquisa realizada pela Organização            acumula em valores altos. Até junho de 2009, cada
das Nações Unidas (ONU), a respeito da quantidade de             aposta de seis dezenas, pertencentes ao conjunto {01,
pessoas com 60 anos ou mais em todo o mundo. Os                  02, 03, ..., 59, 60}, custava R$ 1,50.
números da coluna da direita representam as faixas               Disponível em: www.caixa.gov.br. Acesso em: 7 jul. 2009.

percentuais. Por exemplo, em 1950 havia 95 milhões de
                                                                 Considere que uma pessoa decida apostar exatamente
pessoas com 60 anos ou mais nos países desenvolvidos,
                                                                 R$ 126,00 e que esteja mais interessada em acertar
número entre 10% e 15% da população total nos países
desenvolvidos.                                                   apenas cinco das seis dezenas da mega sena,
                                                                 justamente pela dificuldade desta última. Nesse caso, é
                                                                 melhor que essa pessoa faça 84 apostas de seis
                                                                 dezenas diferentes, que não tenham cinco números em
                                                                 comum, do que uma única aposta com nove dezenas,
                                                                 porque a probabilidade de acertar a quina no segundo
                                                                 caso em relação ao primeiro é, aproximadamente,
                                                                      1                                          1
                                                                 (A) 1 vez menor                       (B) 2       vezes menor
                                                                      2                                          2
                                                                 (C) 4 vezes menor                     (D) 9 vezes menor
                                                                 (E) 14 vezes menor

                                                                 25) (UERJ-2011 -1º ex qualif) Uma fábrica produz
                                                                 sucos com os seguintes sabores: uva, pêssego e
                                                                 laranja. Considere uma caixa com 12 garrafas desses
                                                                 sucos, sendo 4 garrafas de cada sabor.
                                                                 Retirando-se, ao acaso, 2 garrafas dessa caixa, a
                                                                 probabilidade de que ambas contenham suco com o
                                                                 mesmo sabor equivale a:
                                                                 (A) 9,1%               (B) 18,2%
Disponível em: www.economist.com.
Acesso em: 9 jul. 2009 (adaptado).
                                                                 (C) 27,3%              (D) 36,4%

                                                                                                                                       2011
                                                             6
MÓDULO I – PARTE 6                         MATEMÁTICA
                  Projeto
                 Vestibular                      Noções de                                Prof. Bruno Vianna
                                               Probabilidade

26) (UERJ 2011- 2ºex qualif) Uma máquina contém                31) (UFF-2010-2ºF) Dois dados cúbicos não viciados,
pequenas bolas de borracha de 10 cores diferentes,             cujas faces estão numeradas de 1 a 6, são jogados
sendo 10 bolas de cada cor. Ao inserir uma moeda na            aleatoriamente e simultaneamente sobre uma mesa
máquina, uma bola é expelida ao acaso.                         plana. Se a soma dos valores sorteados(*) for um
Observe a ilustração:                                          número par, Paulo ganha a partida. Se a soma for um
                                                               número ímpar, Lúcia ganha. Ao perder a primeira
                                                               partida, Lúcia diz que não irá mais jogar porque a regra
                                                               favorece Paulo. Seu argumento é o seguinte: dentre os
                                                               onze valores possíveis para a soma (os inteiros de 2 a
                                                               12), há seis números pares e apenas cinco números
                                                               ímpares. Logo, Paulo tem maior probabilidade de
Para garantir a retirada de 4 bolas de uma mesma cor, o        ganhar.
menor número de moedas a serem inseridas na
máquina corresponde a:                                         a) Calcule a probabilidade de Lúcia ganhar uma partida.
                                                               Justifique sua resposta.
(A) 5          (B) 13          (C) 31          (D) 40
                                                               b) Use o item a para verificar se o argumento de Lúcia
27) (UERJ 2011- 2ºex qualif) Inserindo-se 3 moedas,            está correto.
uma de cada vez, a probabilidade de que a máquina              (*) Valor sorteado é o número escrito na face do cubo
libere 3 bolas, sendo apenas duas delas brancas, é             oposta à face que está apoiada na mesa.
aproximadamente de:
                                                               32) (PUC-2010 – 2ª f)
(A) 0,008                      (B) 0,025                       Considere o lançamento de três dados comuns.
(C) 0,040                      (D) 0,072
                                                               a) Qual é a probabilidade de que a soma dos valores
28) (UFRJ-2004-PE) Manoel e Joaquim resolveram                 sorteados seja igual a 5?
disputar o seguinte jogo: uma bola será retirada ao
acaso de uma urna que contém 999 bolas idênticas,              b) Qual é a probabilidade de que os três números
numeradas de 1 a 999. Se o número sorteado for par,            sorteados sejam diferentes?
ganha Manoel; se for ímpar Joaquim ganha. Isto foi
resolvido após muita discussão, pois ambos queriam as          33) (UERJ-2011-2ª FASE) Para a realização de uma
pares.                                                         partida de futebol são necessários três árbitros: um juiz
       Se todas as bolas tem a mesma                           principal, que apita o jogo, e seus dois auxiliares, que
probabilidade de serem retiradas, identifique quem             ficam nas laterais. Suponha que esse trio de arbitragem
tem mais chances de ganhar o jogo. Justifique sua              seja escolhido aleatoriamente em um grupo composto
resposta.                                                      de somente dez árbitros, sendo X um deles. Após essa
                                                               escolha, um segundo sorteio aleatório é feito entre os
29) (UFRJ-98-PE) Duzentas bolas pretas e duzentas              três para determinar qual deles será o juiz principal.
bolas brancas são distribuídas em duas urnas, de modo
que cada uma delas contenha cem bolas pretas e cem             Calcule a probabilidade de X ser o juiz principal.
bolas brancas. Uma pessoa retira ao acaso uma bola de
cada urna.                                                     34) (UERJ-2007-ESP) João recorta um círculo de papel
                                                               com 10 cm de raio. Em seguida, dobra esse recorte ao
       Determine a probabilidade de que as duas                meio várias vezes, conforme ilustrado abaixo.
bolas retiradas sejam de cores distintas.

30) (UFRJ-2009) João criou uma senha de 4 algarismos
para o segredo de seu cofre. Mais tarde, quando foi
abrir o cofre, João percebeu que não lembrava mais
qual era a senha, mas sabia que os algarismos eram 1,
3, 8 e 9. Ele, então, resolveu escrever todos os números
possíveis formados pelos 4 algarismos e, em seguida,
tentar abrir o cofre sorteando ao acaso, um a um, os
números de sua lista, sem repetir números já testados.
a) Determine quantos números João escreveu.
b) Calcule a probabilidade de que ele abra o cofre na
12ª tentativa.
                                                                                                                    2011
                                                           7
MÓDULO I – PARTE 6                           MATEMÁTICA
                 Projeto
                Vestibular                     Noções de                              Prof. Bruno Vianna
                                             Probabilidade

Depois de fazer diversas dobras, abre o papel e coloca
o número 1 nas duas extremidades da primeira dobra.
Sucessivamente, no meio de cada um dos arcos
formados pelas dobras anteriores, João escreve a soma
dos números que estão nas extremidades de cada arco.
As figuras a seguir ilustram as quatro etapas iniciais
desse processo.
                                                             Uma criança rasgou algumas cartas desse baralho, e as
                                                             n cartas restantes, não rasgadas, foram guardadas
                                                             em uma caixa.
                                                             A tabela abaixo apresenta as probabilidades de retirar-
                                                             se dessa caixa, ao acaso, as seguintes cartas:




                                                             Calcule o valor de n.

                                                             36) (UERJ-2010-ESP) Uma criança guarda moedas
                                                             de R$ 1,00 e de R$ 0,50 em duas caixas, uma verde
                                                             e outra amarela. Na caixa amarela, há, exatamente,
A figura correspondente à etapa 3 foi colada em uma          12 moedas de R$ 1,00 e 15 moedas de R$ 0,50.
roleta, que após ser girada pode parar, ao acaso, em
apenas oito posições distintas. Uma seta indica o            Admita que, após a transferência de n moedas de R$
número correspondente a cada posição, como ilustra a
                                                             1,00 da caixa verde para a amarela, a probabilidade
figura abaixo.
                                                             de se retirar ao acaso uma moeda de R$ 1,00 da
                                                             caixa amarela seja igual a 50%.

                                                             Calcule o valor de n.

                                                             37) (UFRJ-2010) Um ponto P é aleatoriamente
                                                             selecionado num retângulo S de dimensões 50 cm por
                                                             20 cm. Considere, a partir de S, as seguintes regiões:

                                                             Região A – retângulo de dimensões 15 cm por 4 cm
João girou a roleta duas vezes consecutivas e anotou         com centro no centro de S e
os números indicados pela seta após cada parada.
                                                             Região B – círculo de raio 4 cm com centro no centro de
Calcule a probabilidade de a soma desses números             S.
ser par.
                                                             Suponha que a probabilidade de que o ponto P
35) (UERJ-2009-ESP)                                          pertença a uma região contida em S seja proporcional à
Os baralhos comuns são compostos de 52 cartas                área da região.
divididas em quatro naipes, denominados copas,               Determine a probabilidade de que P pertença
espadas, paus e ouros, com treze cartas distintas de         simultaneamente às regiões A e B.
cada um deles.
Observe a figura que mostra um desses baralhos, no           38) (UFRJ-2011) Um ponto M é selecionado ao acaso
qual as cartas representadas pelas letras A, J, Q e K        no interior de um círculo C de raio 2 e centro O. Em
são denominadas, respectivamente, ás, valete, dama e         seguida, constrói-se um quadrado, também centrado em
rei.                                                         O, que tem M como ponto médio de um de seus lados.

                                                             Calcule a probabilidade de que o quadrado assim
                                                             construído esteja inteiramente contido no círculo C.

                                                                                                               2011
                                                         8
MÓDULO I – PARTE 6                            MATEMÁTICA
                  Projeto
                 Vestibular                       Noções de                                 Prof. Bruno Vianna
                                                Probabilidade

39) (PUC-2010 – 2ª f) O diagrama abaixo mostra uma              40) (UNICAMP - 2002) Em Matemática, um número
sala do jogo Os Labirintos da Simetria. Isaac, o herói do       natural a é chamado palíndromo se seus algarismos,
jogo, entra na sala por um portão no extremo esquerdo           escritos em ordem inversa, produzem o mesmo número.
da sala e precisa sair pelo portão que está no extremo          Por exemplo, 8, 22 e 373 são palíndromos. Pergunta-se:
direito da sala e que inicialmente está fechado.
                                                                a) Quantos números naturais palíndromos existem entre
                                                                1 e 9.999?

                                                                b) Escolhendo-se ao acaso um número natural entre 1 e
                                                                9.999, qual é a probabilidade de que esse número seja
No corredor entre os dois portões há sete cristais, cada        palíndromo? Tal probabilidade é maior ou menor que
um com uma cor do arco íris: Vermelho, Laranja,                 2%? Justifique sua resposta.
Amarelo, Verde, Azul, Índigo e Violeta. A cada partida
as posições dos cristais são sorteadas, com igual               GABARITOS
probabilidade para cada uma das ordens possíveis.
Para que o portão de saída se abra, Isaac precisa tocar         01) B           02) C             03) B             04) A
os sete cristais exatamente na ordem acima. Na sala há
uma corrente de ar da esquerda para a direita. Assim,           05) C           06) B             07) D             08) C
Isaac pode mover-se facilmente da esquerda para a
direita, mas para mover-se da direita para a esquerda           09) A           10) D             11) C             12) D
ele precisa acionar as suas Hélices Mágicas. Cada vez
que ele aciona as Hélices ele gasta uma carga. Para             13) C           14) C             15) A             16) C
tocar um cristal, Isaac deve desligar as Hélices e se
depois de tocar um cristal ele precisar se mover                18) B           19) a) 1/1140     b) 1/570     c) 1/95
novamente para a esquerda ele precisará gastar outra
carga.                                                          20) a) 1/12   b) 5/12             21) D             22) C

Assim, por exemplo, se num jogo a posição dos cristais          23) C           24) C             25) C             26) C
for:
                                                                27) B
Amarelo - Laranja - Índigo - Verde - Violeta - Vermelho –
Azul                                                            28) Joaquim tem mais chances de ganhar o jogo, já que
                                                                há 500 bolas com números ímpares e 499 bolas com
então Isaac chegará gratuitamente ao cristal Vermelho,          números pares.
gastará uma carga para voltar até Laranja e uma
segunda para voltar até Amarelo. Depois disso ele se            29) 50%                   30) a) 24       b) 1/24
moverá gratuitamente até Verde e daí até Azul. Isaac
gastará uma terceira carga para voltar até Índigo e             31) a) 50%    b) 50%      32) a) 1/36     b) 5/9
depois se moverá gratuitamente até Violeta e de lá para
o portão de saída, finalmente aberto. Neste exemplo,            33) 1/10        34) 5/8           35) n = 40
para passar pela sala, Isaac gastou três cargas.
                                                                                          16π + 24 3
Considerando agora uma sala com cristais em posições            36) n = 3       37) P =                             38) ½
sorteadas, responda:                                                                         3000
                                                                39) a) 1/7! = 1/5040      b) 6/7! = 1/840 c) 120/7! = 1/42
a) Qual a probabilidade de que Isaac possa passar pela
sala sem gastar nenhuma carga?                                  40) a) 196    b) 1,96

b) Qual a probabilidade de que Isaac passe pela sala
gastando uma carga para ir de Vermelho até Laranja e
depois não precise gastar mais nenhuma outra carga?

c) Qual a probabilidade de que Isaac precise gastar
exatamente uma carga para passar pela sala?




                                                                                                                         2011
                                                            9
MÓDULO I – PARTE 6                                MATEMÁTICA
                          Projeto
                         Vestibular                 Noções de                                     Prof. Bruno Vianna
                                                  Probabilidade

Algumas resoluções:                                                Questão 30)

Questão 8)                                                         a) São 4 algarismos distintos. Tem-se que 4! = 24. João
                                                                   escreveu 24 números.
             Fla                Tiro                               b)Solução da Banca:
                                                                   Olhando-se uma lista qualquer dos 24 números
         60 - x             x    70 - x                            possíveis, observe que a probabilidade da senha correta
                                                                   estar na n-ésima posição não depende de n. Deste
                                                                   modo a probabilidade de João acertar na 12ª tentativa é
60 – x + x + 70 –x = 100 >> x = 30%                                igual à probabilidade de João acertar na primeira, que é
                                                                   1/24
Como x ≥ 0 >> x ≤ 60%                                              Solução mais simples:
                                                                   Para que João acerte apenas na 12ª tentativa,
Questão 9)                                                         obrigatoriamente ele deve errar as onze tentativas
                                                                   anteriores e acertar a 12ª, logo:
            C 5,1         5 1                                            23 22 21 20 19 18 17 16 15 14 13 1
   Resol:            =     =                                       P=      ⋅ ⋅ ⋅ ⋅ ⋅ ⋅ ⋅ ⋅ ⋅ ⋅ ⋅
            C 7 ,3       35 7                                            24 23 22 21 20 19 18 17 16 15 14 13


As cinco são : João Pedro __ ____ ____ ____ ____
                                                                                                      1
                                                                                                P=
Questão 19)                                                                                           24
Há C20,3 = 1140 maneiras de se retirarem 3 bolas da                Questão 31)
urna.
                                                                   a) O espaço amostral desse experimento é o conjunto
a) Soma igual a 6: 1 + 2 + 3 (somente um maneira).                 A, com 36 elementos:
Logo P(a) = 1/1140.
                                                                   A = { (1, 1), (1, 2), (1, 3), (1, 4), (1, 5), (1, 6),(2, 1), (2, 2),
b) Soma igual a 8: 1 + 2 + 5 e 1 + 3 + 4 (duas                     (2, 3), (2, 4), (2, 5), (2, 6),(3, 1), (3, 2), (3, 3), (3, 4), (3,
maneiras). Logo P (b) = 2/1140 = 1/570.                            5), (3, 6),(4, 1), (4, 2), (4, 3), (4, 4), (4, 5), (4, 6),(5, 1),
                                                                   (5, 2), (5, 3), (5, 4), (5, 5), (5, 6),(6, 1), (6, 2), (6, 3), (6,
c) Soma igual a 15:                                                4), (6, 5), (6, 6) }.
1 + 2 + 12, 1 + 3 + 11, 1 + 4 + 10, 1 + 5 + 9, 1 + 6 + 8, 2
+ 3 + 10, 2 + 4 + 9,                                               O evento “a soma dos valores sorteados é um número
2 + 5 + 8, 2 + 6 + 7, 3 + 4 + 8, 3 + 5 + 7, 4 + 5 + 6 (doze        ímpar” é o conjunto E, com 18 elementos:
maneiras). Logo P (c) = 12/1140 = 1/95.
                                                                   E = { (1, 2), (1, 4), (1, 6),(2, 1), (2, 3), (2, 5),(3, 2), (3, 4),
Questão 20)                                                        (3, 6),(4, 1), (4, 3), (4, 5),(5, 2), (5, 4), (5, 6),(6, 1), (6, 3),
Há C10,3 =120 maneiras de se retirarem 3 bolas da                  (6, 5) }.
urna.
                                                                   Logo, a probabilidade de Lúcia ganhar é igual a 18/36 =
                                     C      10   1
a) Tirar três bolas brancas: P (a ) = 5,3 =    =                   1/2 = 50%.
                                     C10,3 120 12
                                                                   b) O cálculo feito no item (a) mostra que Paulo e Lúcia
b) Tirar duas brancas e uma preta:                                 têm a mesma probabilidade de ganhar uma partida.
        C ⋅C         50    5                                       Questão 32)
P(b) = 5, 2 5,1 =        =                                         Temos no lançamento de três dados 63 possibilidades.
          C10,3     120 12
                                                                   a) O evento ter soma 5, tem casos : (1,2,2), (2,2,1) ,
Questão 29) Qualquer que seja a cor da bola retirada               (1,2,1),(1,1,3),(1,3,1) e (3,1,1) então,
na primeira urna, a chance de se retirar uma bola de cor           P= 6/216 = 1/36
diferente da segunda urna é de 100/200.
                                                                   b) O evento ter todos os números diferentes, vale 6 × 5
Logo: P = ½ = 50%                                                  × 4. Logo, P = (6.5.4)/216 = 5/9

                                                                                                                                 2011
                                                              10
MÓDULO I – PARTE 6                      MATEMÁTICA
                       Projeto
                      Vestibular                       Noções de                           Prof. Bruno Vianna
                                                     Probabilidade

Questão 33)
A = {x; a2 ; a3 ; a4 ; ...; a10}

                             9 ⋅8
                    C9, 2
                          = 2 ⋅1 =
                                      36   3
1º sorteio: P( x) =                      =
                    C10,3 10 ⋅ 9 ⋅ 8 120 10
                            3 ⋅ 2 ⋅1
            1 3      1                                         Observando-se o triângulo retângulo OLN, tem-se que o
P( xJUÍZ ) = ⋅ =                                               ângulo LÔN mede 60º. Assim, a medida da área do
            3 10 10
                                                                                       8π 2
                                                               setor circular OMN é       cm e a área do triângulo OLN
Questão 34)                                                                             3
                                                1              é 2 3 cm .
                                                                         2
Probabilidade de ocorrer par e par ⇒ P1 =
                                                16             Portanto, a medida da área da região C
Probabilidade de ocorrer ímpar e ímpar ⇒ P2 =
                                                     9           8π       
                                                     16
                                                               é    − 2 3  cm2 .
                                                                 3        
Probabilidade de ocorrer soma par ⇒                            Logo, a medida da área de A∩ B é:
          10   5
P1 + P2 =    =                                                         8π       2
          16   8                                               16π − 4 3 − 2 3  cm
                                                                               
                                                                                                           2
Questão 35)                                                    Como a medida da área de S é 1000 cm , tem-se que a
                                                                                                   16π + 24 3
Número de cartas guardadas na caixa: n                         probabilidade solicitada é P =                 .
                                                                                                      3000
Probabilidade de retirada de:
- um rei → P(R) = 0,075
- uma carta de copas → P(C) = 0,25                             Questão 38)
- um carta de copas ou um rei → P(C ∩ R) = 0,3                 A diagonal do quadrado inscrito é igual ao diâmetro do
- o rei de copas → P(R ∩ C) = P(R) + P(C) – P(R ∪ C)           círculo C, ou seja, d = 4. A medida L do lado deste
                                                                                               2
                                                               quadrado é, por Pitágoras, 2L = 16 , ou seja, L = 2 2 .
                                            1
P(R ∩ C) = 0,075 + 0,25 − 0,3 = 0,025 =
                                            n
1 1
 =   ⇒ n = 40
n 40
Questão 36)
                1   12 + n
                  =                                            Para que o quadrado esteja inteiramente contido em C,
                2 12 + n + 15                                  a distância de M ao centro de C deve ser menor do que
→ 12 + n + 15 = 2 (12 + n) →                                                 L
→      n + 27 = 24 + 2n →                                      ou igual a      . Ou seja, M pertence a um círculo CM de
→    27 – 24 = 2n – n →                                                      2
→          n=3                                                        L
                                                               raio     e mesmo centro C.
                                                                      2
Questão 37)
Por hipótese, a probabilidade de que o ponto P pertença        Então a probabilidade pedida é:
a uma região F, contida em S, é dada pela razão entre a
medida da área de F e a medida da área de S.
                                                                      área (CM ) 2π 1
Assim, a probabilidade de que o ponto P pertença a             p=               =  =
                                   área ( A ∩ B)                       área (C ) 4π 2
ambas as regiões é dada por:
                                     área ( S )
Seja C a região sombreada na figura abaixo. Então,a
área (A∩B) = 16π – 4 × área (C).



                                                                                                                    2011
                                                          11
MÓDULO I – PARTE 6      MATEMÁTICA
                   Projeto
                  Vestibular                         Noções de      Prof. Bruno Vianna
                                                   Probabilidade

Questão 39)

a) Isto só ocorrerá se os cristais estiverem na ordem:

Vermelho - Laranja - Amarelo - Verde - Azul - Índigo -
Violeta

A probabilidade de isso ocorrer é 1/7! = 1/5040.

b) Isto ocorrerá se as cores:

Laranja - Amarelo - Verde - Azul - Índigo – Violeta

aparecerem nesta ordem da esquerda para a direita,
com Vermelho em qualquer posição exceto na primeira.
Há, assim, 6 configurações possíveis e a probabilidade
pedida é 6/7! = 1/840.

c) Para formar uma configuração deste tipo, devemos
                                                 7
primeiro selecionar um conjunto de posições (há 2 =
128 maneiras de fazer isso).

Primeiro preenchemos as posições do conjunto da
esquerda para a direita com as cores na ordem em que
Isaac deve tocá-las e depois preenchemos as posições
no complemento do conjunto. Isto só *não* funcionará
se as posições do conjunto estiverem todas à esquerda
das posições do complemento (pois neste caso Isaac
não gastaria nenhuma carga), ou seja, para os 8
conjuntos {}, {1}, {1,2}, {1,2,3}, ..., {1,2,3,4,5,6,7}.

Assim há 128 - 8 = 120 configurações possíveis, e a
probabilidade pedida é 120/7! = 1/42.

Questão 40)

a) De 1 até 9.999, temos desde palíndromos de 1
algarismo até palíndromos de 4 algarismos.
    Assim,
               x          x            x x
        ou         ou         ou

      9 + 9 ⋅ 1 + 9 ⋅ 10 ⋅ 1 + 9 ⋅ 10 ⋅ 1 ⋅ 1 = 198
    Considerando que “entre 1 e 9.999” não devam ser
    incluídos os extremos, temos 196 palíndromos.
    Resposta: 196

b) “Entre 1 e 9.999” temos 9.997 números.
    Assim, a probabilidade pedida é:
           196
    P=           ≈ 1,96 %
          9.997
Nota: Se interpretássemos o “entre 1 e 9.999” com a
possibilidade da inclusão dos extremos, teríamos:
a) 198 palíndromos.
         198      2
b) P =         =      ≈ 1,98 %.
        9.999 101


                                                                                         2011
                                                           12

Más contenido relacionado

La actualidad más candente

Aula 02 polígonos - exercicios
Aula 02   polígonos - exerciciosAula 02   polígonos - exercicios
Aula 02 polígonos - exerciciosJeane Carvalho
 
Funcões Injetora, Sobrejetora e Bijetora
Funcões Injetora, Sobrejetora e BijetoraFuncões Injetora, Sobrejetora e Bijetora
Funcões Injetora, Sobrejetora e BijetoraCleiton Cunha
 
Lista de exercícios – sistema de equações do 1° grau
Lista de exercícios – sistema de equações do 1° grauLista de exercícios – sistema de equações do 1° grau
Lista de exercícios – sistema de equações do 1° grauEverton Moraes
 
Lista de exercícios equação - 7 ano - 5ª etapa
Lista de exercícios   equação - 7 ano - 5ª etapaLista de exercícios   equação - 7 ano - 5ª etapa
Lista de exercícios equação - 7 ano - 5ª etapaLuciana Ayres
 
Numeros inteiros piramide para o slide
Numeros inteiros piramide para o slideNumeros inteiros piramide para o slide
Numeros inteiros piramide para o slideAdriano Augusto
 
Revisão de geometria 6º ano - Áreas e medidas de superfície
Revisão de geometria   6º ano - Áreas e medidas de superfícieRevisão de geometria   6º ano - Áreas e medidas de superfície
Revisão de geometria 6º ano - Áreas e medidas de superfícierabillamat1
 
1ª lista de exercícios análise de gráficos e porcentagem
1ª lista de exercícios   análise de gráficos e porcentagem1ª lista de exercícios   análise de gráficos e porcentagem
1ª lista de exercícios análise de gráficos e porcentagemlualvares
 
Lista de exercícios de função afim
Lista de exercícios de função afimLista de exercícios de função afim
Lista de exercícios de função afimProfessoraIve
 
FunçOes Injetoras, Sobrejetoras E Sobrejetoras
FunçOes Injetoras, Sobrejetoras E SobrejetorasFunçOes Injetoras, Sobrejetoras E Sobrejetoras
FunçOes Injetoras, Sobrejetoras E Sobrejetorasandreabelchol
 
prof.Calazans(Geom.plana) - Polígonos(20 questões resolvidas)
prof.Calazans(Geom.plana) - Polígonos(20 questões resolvidas)prof.Calazans(Geom.plana) - Polígonos(20 questões resolvidas)
prof.Calazans(Geom.plana) - Polígonos(20 questões resolvidas)ProfCalazans
 
Lista de Exercícios – Razão e Proporção
Lista de Exercícios – Razão e ProporçãoLista de Exercícios – Razão e Proporção
Lista de Exercícios – Razão e ProporçãoEverton Moraes
 
Exercícios de Potência - 2011
Exercícios de Potência - 2011Exercícios de Potência - 2011
Exercícios de Potência - 2011tioheraclito
 
Função afim-linear-constante-gráficos
Função  afim-linear-constante-gráficosFunção  afim-linear-constante-gráficos
Função afim-linear-constante-gráficosmarmorei
 
Sistemas Lineares 2equacoes 2incognitas
Sistemas Lineares 2equacoes 2incognitasSistemas Lineares 2equacoes 2incognitas
Sistemas Lineares 2equacoes 2incognitastioheraclito
 
3º ano 2ª lista de exercícios poliedros - prismas
3º ano   2ª lista de exercícios   poliedros - prismas3º ano   2ª lista de exercícios   poliedros - prismas
3º ano 2ª lista de exercícios poliedros - prismasTiago Silva
 
Trigonometria no Triângulo Retângulo 2011
Trigonometria no Triângulo Retângulo 2011Trigonometria no Triângulo Retângulo 2011
Trigonometria no Triângulo Retângulo 2011tioheraclito
 
Exercícios monomios extra 8º ano
Exercícios monomios extra   8º anoExercícios monomios extra   8º ano
Exercícios monomios extra 8º anoAdriano Capilupe
 

La actualidad más candente (20)

Aula 02 polígonos - exercicios
Aula 02   polígonos - exerciciosAula 02   polígonos - exercicios
Aula 02 polígonos - exercicios
 
Funcões Injetora, Sobrejetora e Bijetora
Funcões Injetora, Sobrejetora e BijetoraFuncões Injetora, Sobrejetora e Bijetora
Funcões Injetora, Sobrejetora e Bijetora
 
Lista de exercícios – sistema de equações do 1° grau
Lista de exercícios – sistema de equações do 1° grauLista de exercícios – sistema de equações do 1° grau
Lista de exercícios – sistema de equações do 1° grau
 
Situação problemas ideia de função
Situação problemas   ideia de funçãoSituação problemas   ideia de função
Situação problemas ideia de função
 
Lista de exercícios equação - 7 ano - 5ª etapa
Lista de exercícios   equação - 7 ano - 5ª etapaLista de exercícios   equação - 7 ano - 5ª etapa
Lista de exercícios equação - 7 ano - 5ª etapa
 
Função afim
Função afimFunção afim
Função afim
 
Numeros inteiros piramide para o slide
Numeros inteiros piramide para o slideNumeros inteiros piramide para o slide
Numeros inteiros piramide para o slide
 
Revisão de geometria 6º ano - Áreas e medidas de superfície
Revisão de geometria   6º ano - Áreas e medidas de superfícieRevisão de geometria   6º ano - Áreas e medidas de superfície
Revisão de geometria 6º ano - Áreas e medidas de superfície
 
1ª lista de exercícios análise de gráficos e porcentagem
1ª lista de exercícios   análise de gráficos e porcentagem1ª lista de exercícios   análise de gráficos e porcentagem
1ª lista de exercícios análise de gráficos e porcentagem
 
Lista de exercícios de função afim
Lista de exercícios de função afimLista de exercícios de função afim
Lista de exercícios de função afim
 
FunçOes Injetoras, Sobrejetoras E Sobrejetoras
FunçOes Injetoras, Sobrejetoras E SobrejetorasFunçOes Injetoras, Sobrejetoras E Sobrejetoras
FunçOes Injetoras, Sobrejetoras E Sobrejetoras
 
prof.Calazans(Geom.plana) - Polígonos(20 questões resolvidas)
prof.Calazans(Geom.plana) - Polígonos(20 questões resolvidas)prof.Calazans(Geom.plana) - Polígonos(20 questões resolvidas)
prof.Calazans(Geom.plana) - Polígonos(20 questões resolvidas)
 
Lista de Exercícios – Razão e Proporção
Lista de Exercícios – Razão e ProporçãoLista de Exercícios – Razão e Proporção
Lista de Exercícios – Razão e Proporção
 
Exercícios de Potência - 2011
Exercícios de Potência - 2011Exercícios de Potência - 2011
Exercícios de Potência - 2011
 
Função afim-linear-constante-gráficos
Função  afim-linear-constante-gráficosFunção  afim-linear-constante-gráficos
Função afim-linear-constante-gráficos
 
Zero da função do 1º grau
Zero da função do 1º grauZero da função do 1º grau
Zero da função do 1º grau
 
Sistemas Lineares 2equacoes 2incognitas
Sistemas Lineares 2equacoes 2incognitasSistemas Lineares 2equacoes 2incognitas
Sistemas Lineares 2equacoes 2incognitas
 
3º ano 2ª lista de exercícios poliedros - prismas
3º ano   2ª lista de exercícios   poliedros - prismas3º ano   2ª lista de exercícios   poliedros - prismas
3º ano 2ª lista de exercícios poliedros - prismas
 
Trigonometria no Triângulo Retângulo 2011
Trigonometria no Triângulo Retângulo 2011Trigonometria no Triângulo Retângulo 2011
Trigonometria no Triângulo Retângulo 2011
 
Exercícios monomios extra 8º ano
Exercícios monomios extra   8º anoExercícios monomios extra   8º ano
Exercícios monomios extra 8º ano
 

Similar a Probabilidades e eventos

Similar a Probabilidades e eventos (20)

Aula 22 probabilidade - parte 1
Aula 22   probabilidade - parte 1Aula 22   probabilidade - parte 1
Aula 22 probabilidade - parte 1
 
aula 02 Teorema de bayes, demonstração..
aula 02 Teorema de bayes, demonstração..aula 02 Teorema de bayes, demonstração..
aula 02 Teorema de bayes, demonstração..
 
Aula 5 probabilidade
Aula 5   probabilidadeAula 5   probabilidade
Aula 5 probabilidade
 
Teoria_das_Probabilidades_processos_estocasticos.pptx
Teoria_das_Probabilidades_processos_estocasticos.pptxTeoria_das_Probabilidades_processos_estocasticos.pptx
Teoria_das_Probabilidades_processos_estocasticos.pptx
 
04 probabilidade
04 probabilidade04 probabilidade
04 probabilidade
 
Estatística: Modelos Discretos
Estatística: Modelos DiscretosEstatística: Modelos Discretos
Estatística: Modelos Discretos
 
Probabilidade e Estatística - Aula 04
Probabilidade e Estatística - Aula 04Probabilidade e Estatística - Aula 04
Probabilidade e Estatística - Aula 04
 
Apostila probabilidade
Apostila probabilidadeApostila probabilidade
Apostila probabilidade
 
Probabilidade
ProbabilidadeProbabilidade
Probabilidade
 
Probabilidades - parte 2 (ISMT)
Probabilidades - parte 2 (ISMT)Probabilidades - parte 2 (ISMT)
Probabilidades - parte 2 (ISMT)
 
informações sobre as probabilidades.docx
informações sobre as probabilidades.docxinformações sobre as probabilidades.docx
informações sobre as probabilidades.docx
 
Resumo1
Resumo1Resumo1
Resumo1
 
Probabilidade
ProbabilidadeProbabilidade
Probabilidade
 
Probabilidade Básica
Probabilidade BásicaProbabilidade Básica
Probabilidade Básica
 
Probabilidade
ProbabilidadeProbabilidade
Probabilidade
 
Probabilidade 150526011133-lva1-app6891
Probabilidade 150526011133-lva1-app6891Probabilidade 150526011133-lva1-app6891
Probabilidade 150526011133-lva1-app6891
 
Teoria de Probabilidades-2021.pptx
Teoria de  Probabilidades-2021.pptxTeoria de  Probabilidades-2021.pptx
Teoria de Probabilidades-2021.pptx
 
Probabilidade - Definição
Probabilidade - DefiniçãoProbabilidade - Definição
Probabilidade - Definição
 
Probabilidade - Definição
Probabilidade - DefiniçãoProbabilidade - Definição
Probabilidade - Definição
 
Probabilidade u(1)
Probabilidade u(1)Probabilidade u(1)
Probabilidade u(1)
 

Más de con_seguir

Transformações geométricas no plano
Transformações geométricas no planoTransformações geométricas no plano
Transformações geométricas no planocon_seguir
 
Sistemas lineares
Sistemas linearesSistemas lineares
Sistemas linearescon_seguir
 
Relações métricas no triângulo retângulo
Relações métricas no triângulo retânguloRelações métricas no triângulo retângulo
Relações métricas no triângulo retângulocon_seguir
 
Numeros complexos aula
Numeros complexos aulaNumeros complexos aula
Numeros complexos aulacon_seguir
 
Numeros complexos
Numeros complexosNumeros complexos
Numeros complexoscon_seguir
 
Matematica raciocinio logico
Matematica raciocinio logicoMatematica raciocinio logico
Matematica raciocinio logicocon_seguir
 
Matematica questões resolvidas i
Matematica questões resolvidas iMatematica questões resolvidas i
Matematica questões resolvidas icon_seguir
 
Geometria analitica exercicios resolvidos
Geometria analitica exercicios resolvidosGeometria analitica exercicios resolvidos
Geometria analitica exercicios resolvidoscon_seguir
 
Geometria analitica equacao da reta
Geometria analitica equacao da retaGeometria analitica equacao da reta
Geometria analitica equacao da retacon_seguir
 
Fundamentos matematica iv
Fundamentos matematica ivFundamentos matematica iv
Fundamentos matematica ivcon_seguir
 
Fundamentos matematica ii
Fundamentos matematica iiFundamentos matematica ii
Fundamentos matematica iicon_seguir
 
Fundamentos matematica i
Fundamentos matematica iFundamentos matematica i
Fundamentos matematica icon_seguir
 
Fundamentos geometria i
Fundamentos geometria iFundamentos geometria i
Fundamentos geometria icon_seguir
 
Funcao do primeiro grau
Funcao do primeiro grauFuncao do primeiro grau
Funcao do primeiro graucon_seguir
 
Fisica 003 optica
Fisica   003 opticaFisica   003 optica
Fisica 003 opticacon_seguir
 
Exercicios resolvidos poligonos
Exercicios resolvidos   poligonosExercicios resolvidos   poligonos
Exercicios resolvidos poligonoscon_seguir
 
Estudos da reta
Estudos da retaEstudos da reta
Estudos da retacon_seguir
 

Más de con_seguir (20)

Transformações geométricas no plano
Transformações geométricas no planoTransformações geométricas no plano
Transformações geométricas no plano
 
Sistemas lineares
Sistemas linearesSistemas lineares
Sistemas lineares
 
Relações métricas no triângulo retângulo
Relações métricas no triângulo retânguloRelações métricas no triângulo retângulo
Relações métricas no triângulo retângulo
 
Ponto reta
Ponto retaPonto reta
Ponto reta
 
Poliedro
PoliedroPoliedro
Poliedro
 
Numeros complexos aula
Numeros complexos aulaNumeros complexos aula
Numeros complexos aula
 
Numeros complexos
Numeros complexosNumeros complexos
Numeros complexos
 
Matematica raciocinio logico
Matematica raciocinio logicoMatematica raciocinio logico
Matematica raciocinio logico
 
Matematica questões resolvidas i
Matematica questões resolvidas iMatematica questões resolvidas i
Matematica questões resolvidas i
 
Geometria analitica exercicios resolvidos
Geometria analitica exercicios resolvidosGeometria analitica exercicios resolvidos
Geometria analitica exercicios resolvidos
 
Geometria analitica equacao da reta
Geometria analitica equacao da retaGeometria analitica equacao da reta
Geometria analitica equacao da reta
 
Geometria
GeometriaGeometria
Geometria
 
Fundamentos matematica iv
Fundamentos matematica ivFundamentos matematica iv
Fundamentos matematica iv
 
Fundamentos matematica ii
Fundamentos matematica iiFundamentos matematica ii
Fundamentos matematica ii
 
Fundamentos matematica i
Fundamentos matematica iFundamentos matematica i
Fundamentos matematica i
 
Fundamentos geometria i
Fundamentos geometria iFundamentos geometria i
Fundamentos geometria i
 
Funcao do primeiro grau
Funcao do primeiro grauFuncao do primeiro grau
Funcao do primeiro grau
 
Fisica 003 optica
Fisica   003 opticaFisica   003 optica
Fisica 003 optica
 
Exercicios resolvidos poligonos
Exercicios resolvidos   poligonosExercicios resolvidos   poligonos
Exercicios resolvidos poligonos
 
Estudos da reta
Estudos da retaEstudos da reta
Estudos da reta
 

Último

Slides Lição 6, CPAD, As Nossas Armas Espirituais, 2Tr24.pptx
Slides Lição 6, CPAD, As Nossas Armas Espirituais, 2Tr24.pptxSlides Lição 6, CPAD, As Nossas Armas Espirituais, 2Tr24.pptx
Slides Lição 6, CPAD, As Nossas Armas Espirituais, 2Tr24.pptxLuizHenriquedeAlmeid6
 
Apresentação em Powerpoint do Bioma Catinga.pptx
Apresentação em Powerpoint do Bioma Catinga.pptxApresentação em Powerpoint do Bioma Catinga.pptx
Apresentação em Powerpoint do Bioma Catinga.pptxLusGlissonGud
 
Bloco de português com artigo de opinião 8º A, B 3.docx
Bloco de português com artigo de opinião 8º A, B 3.docxBloco de português com artigo de opinião 8º A, B 3.docx
Bloco de português com artigo de opinião 8º A, B 3.docxkellyneamaral
 
Recomposiçao em matematica 1 ano 2024 - ESTUDANTE 1ª série.pdf
Recomposiçao em matematica 1 ano 2024 - ESTUDANTE 1ª série.pdfRecomposiçao em matematica 1 ano 2024 - ESTUDANTE 1ª série.pdf
Recomposiçao em matematica 1 ano 2024 - ESTUDANTE 1ª série.pdfFrancisco Márcio Bezerra Oliveira
 
421243121-Apostila-Ensino-Religioso-Do-1-ao-5-ano.pdf
421243121-Apostila-Ensino-Religioso-Do-1-ao-5-ano.pdf421243121-Apostila-Ensino-Religioso-Do-1-ao-5-ano.pdf
421243121-Apostila-Ensino-Religioso-Do-1-ao-5-ano.pdfLeloIurk1
 
Atividade - Letra da música Esperando na Janela.
Atividade -  Letra da música Esperando na Janela.Atividade -  Letra da música Esperando na Janela.
Atividade - Letra da música Esperando na Janela.Mary Alvarenga
 
Discurso Direto, Indireto e Indireto Livre.pptx
Discurso Direto, Indireto e Indireto Livre.pptxDiscurso Direto, Indireto e Indireto Livre.pptx
Discurso Direto, Indireto e Indireto Livre.pptxferreirapriscilla84
 
PROVA - ESTUDO CONTEMPORÂNEO E TRANSVERSAL: COMUNICAÇÃO ASSERTIVA E INTERPESS...
PROVA - ESTUDO CONTEMPORÂNEO E TRANSVERSAL: COMUNICAÇÃO ASSERTIVA E INTERPESS...PROVA - ESTUDO CONTEMPORÂNEO E TRANSVERSAL: COMUNICAÇÃO ASSERTIVA E INTERPESS...
PROVA - ESTUDO CONTEMPORÂNEO E TRANSVERSAL: COMUNICAÇÃO ASSERTIVA E INTERPESS...azulassessoria9
 
COMPETÊNCIA 2 da redação do enem prodção textual professora vanessa cavalcante
COMPETÊNCIA 2 da redação do enem prodção textual professora vanessa cavalcanteCOMPETÊNCIA 2 da redação do enem prodção textual professora vanessa cavalcante
COMPETÊNCIA 2 da redação do enem prodção textual professora vanessa cavalcanteVanessaCavalcante37
 
Projeto_de_Extensão_Agronomia_adquira_ja_(91)_98764-0830.pdf
Projeto_de_Extensão_Agronomia_adquira_ja_(91)_98764-0830.pdfProjeto_de_Extensão_Agronomia_adquira_ja_(91)_98764-0830.pdf
Projeto_de_Extensão_Agronomia_adquira_ja_(91)_98764-0830.pdfHELENO FAVACHO
 
Historia da Arte europeia e não só. .pdf
Historia da Arte europeia e não só. .pdfHistoria da Arte europeia e não só. .pdf
Historia da Arte europeia e não só. .pdfEmanuel Pio
 
o ciclo do contato Jorge Ponciano Ribeiro.pdf
o ciclo do contato Jorge Ponciano Ribeiro.pdfo ciclo do contato Jorge Ponciano Ribeiro.pdf
o ciclo do contato Jorge Ponciano Ribeiro.pdfCamillaBrito19
 
DeClara n.º 75 Abril 2024 - O Jornal digital do Agrupamento de Escolas Clara ...
DeClara n.º 75 Abril 2024 - O Jornal digital do Agrupamento de Escolas Clara ...DeClara n.º 75 Abril 2024 - O Jornal digital do Agrupamento de Escolas Clara ...
DeClara n.º 75 Abril 2024 - O Jornal digital do Agrupamento de Escolas Clara ...IsabelPereira2010
 
Rota das Ribeiras Camp, Projeto Nós Propomos!
Rota das Ribeiras Camp, Projeto Nós Propomos!Rota das Ribeiras Camp, Projeto Nós Propomos!
Rota das Ribeiras Camp, Projeto Nós Propomos!Ilda Bicacro
 
INTERVENÇÃO PARÁ - Formação de Professor
INTERVENÇÃO PARÁ - Formação de ProfessorINTERVENÇÃO PARÁ - Formação de Professor
INTERVENÇÃO PARÁ - Formação de ProfessorEdvanirCosta
 
PRÁTICAS PEDAGÓGICAS GESTÃO DA APRENDIZAGEM
PRÁTICAS PEDAGÓGICAS GESTÃO DA APRENDIZAGEMPRÁTICAS PEDAGÓGICAS GESTÃO DA APRENDIZAGEM
PRÁTICAS PEDAGÓGICAS GESTÃO DA APRENDIZAGEMHELENO FAVACHO
 
5 bloco 7 ano - Ensino Relogioso- Lideres Religiosos _ Passei Direto.pdf
5 bloco 7 ano - Ensino Relogioso- Lideres Religiosos _ Passei Direto.pdf5 bloco 7 ano - Ensino Relogioso- Lideres Religiosos _ Passei Direto.pdf
5 bloco 7 ano - Ensino Relogioso- Lideres Religiosos _ Passei Direto.pdfLeloIurk1
 
Construção (C)erta - Nós Propomos! Sertã
Construção (C)erta - Nós Propomos! SertãConstrução (C)erta - Nós Propomos! Sertã
Construção (C)erta - Nós Propomos! SertãIlda Bicacro
 
Teoria heterotrófica e autotrófica dos primeiros seres vivos..pptx
Teoria heterotrófica e autotrófica dos primeiros seres vivos..pptxTeoria heterotrófica e autotrófica dos primeiros seres vivos..pptx
Teoria heterotrófica e autotrófica dos primeiros seres vivos..pptxTailsonSantos1
 
PROVA - ESTUDO CONTEMPORÂNEO E TRANSVERSAL: LEITURA DE IMAGENS, GRÁFICOS E MA...
PROVA - ESTUDO CONTEMPORÂNEO E TRANSVERSAL: LEITURA DE IMAGENS, GRÁFICOS E MA...PROVA - ESTUDO CONTEMPORÂNEO E TRANSVERSAL: LEITURA DE IMAGENS, GRÁFICOS E MA...
PROVA - ESTUDO CONTEMPORÂNEO E TRANSVERSAL: LEITURA DE IMAGENS, GRÁFICOS E MA...azulassessoria9
 

Último (20)

Slides Lição 6, CPAD, As Nossas Armas Espirituais, 2Tr24.pptx
Slides Lição 6, CPAD, As Nossas Armas Espirituais, 2Tr24.pptxSlides Lição 6, CPAD, As Nossas Armas Espirituais, 2Tr24.pptx
Slides Lição 6, CPAD, As Nossas Armas Espirituais, 2Tr24.pptx
 
Apresentação em Powerpoint do Bioma Catinga.pptx
Apresentação em Powerpoint do Bioma Catinga.pptxApresentação em Powerpoint do Bioma Catinga.pptx
Apresentação em Powerpoint do Bioma Catinga.pptx
 
Bloco de português com artigo de opinião 8º A, B 3.docx
Bloco de português com artigo de opinião 8º A, B 3.docxBloco de português com artigo de opinião 8º A, B 3.docx
Bloco de português com artigo de opinião 8º A, B 3.docx
 
Recomposiçao em matematica 1 ano 2024 - ESTUDANTE 1ª série.pdf
Recomposiçao em matematica 1 ano 2024 - ESTUDANTE 1ª série.pdfRecomposiçao em matematica 1 ano 2024 - ESTUDANTE 1ª série.pdf
Recomposiçao em matematica 1 ano 2024 - ESTUDANTE 1ª série.pdf
 
421243121-Apostila-Ensino-Religioso-Do-1-ao-5-ano.pdf
421243121-Apostila-Ensino-Religioso-Do-1-ao-5-ano.pdf421243121-Apostila-Ensino-Religioso-Do-1-ao-5-ano.pdf
421243121-Apostila-Ensino-Religioso-Do-1-ao-5-ano.pdf
 
Atividade - Letra da música Esperando na Janela.
Atividade -  Letra da música Esperando na Janela.Atividade -  Letra da música Esperando na Janela.
Atividade - Letra da música Esperando na Janela.
 
Discurso Direto, Indireto e Indireto Livre.pptx
Discurso Direto, Indireto e Indireto Livre.pptxDiscurso Direto, Indireto e Indireto Livre.pptx
Discurso Direto, Indireto e Indireto Livre.pptx
 
PROVA - ESTUDO CONTEMPORÂNEO E TRANSVERSAL: COMUNICAÇÃO ASSERTIVA E INTERPESS...
PROVA - ESTUDO CONTEMPORÂNEO E TRANSVERSAL: COMUNICAÇÃO ASSERTIVA E INTERPESS...PROVA - ESTUDO CONTEMPORÂNEO E TRANSVERSAL: COMUNICAÇÃO ASSERTIVA E INTERPESS...
PROVA - ESTUDO CONTEMPORÂNEO E TRANSVERSAL: COMUNICAÇÃO ASSERTIVA E INTERPESS...
 
COMPETÊNCIA 2 da redação do enem prodção textual professora vanessa cavalcante
COMPETÊNCIA 2 da redação do enem prodção textual professora vanessa cavalcanteCOMPETÊNCIA 2 da redação do enem prodção textual professora vanessa cavalcante
COMPETÊNCIA 2 da redação do enem prodção textual professora vanessa cavalcante
 
Projeto_de_Extensão_Agronomia_adquira_ja_(91)_98764-0830.pdf
Projeto_de_Extensão_Agronomia_adquira_ja_(91)_98764-0830.pdfProjeto_de_Extensão_Agronomia_adquira_ja_(91)_98764-0830.pdf
Projeto_de_Extensão_Agronomia_adquira_ja_(91)_98764-0830.pdf
 
Historia da Arte europeia e não só. .pdf
Historia da Arte europeia e não só. .pdfHistoria da Arte europeia e não só. .pdf
Historia da Arte europeia e não só. .pdf
 
o ciclo do contato Jorge Ponciano Ribeiro.pdf
o ciclo do contato Jorge Ponciano Ribeiro.pdfo ciclo do contato Jorge Ponciano Ribeiro.pdf
o ciclo do contato Jorge Ponciano Ribeiro.pdf
 
DeClara n.º 75 Abril 2024 - O Jornal digital do Agrupamento de Escolas Clara ...
DeClara n.º 75 Abril 2024 - O Jornal digital do Agrupamento de Escolas Clara ...DeClara n.º 75 Abril 2024 - O Jornal digital do Agrupamento de Escolas Clara ...
DeClara n.º 75 Abril 2024 - O Jornal digital do Agrupamento de Escolas Clara ...
 
Rota das Ribeiras Camp, Projeto Nós Propomos!
Rota das Ribeiras Camp, Projeto Nós Propomos!Rota das Ribeiras Camp, Projeto Nós Propomos!
Rota das Ribeiras Camp, Projeto Nós Propomos!
 
INTERVENÇÃO PARÁ - Formação de Professor
INTERVENÇÃO PARÁ - Formação de ProfessorINTERVENÇÃO PARÁ - Formação de Professor
INTERVENÇÃO PARÁ - Formação de Professor
 
PRÁTICAS PEDAGÓGICAS GESTÃO DA APRENDIZAGEM
PRÁTICAS PEDAGÓGICAS GESTÃO DA APRENDIZAGEMPRÁTICAS PEDAGÓGICAS GESTÃO DA APRENDIZAGEM
PRÁTICAS PEDAGÓGICAS GESTÃO DA APRENDIZAGEM
 
5 bloco 7 ano - Ensino Relogioso- Lideres Religiosos _ Passei Direto.pdf
5 bloco 7 ano - Ensino Relogioso- Lideres Religiosos _ Passei Direto.pdf5 bloco 7 ano - Ensino Relogioso- Lideres Religiosos _ Passei Direto.pdf
5 bloco 7 ano - Ensino Relogioso- Lideres Religiosos _ Passei Direto.pdf
 
Construção (C)erta - Nós Propomos! Sertã
Construção (C)erta - Nós Propomos! SertãConstrução (C)erta - Nós Propomos! Sertã
Construção (C)erta - Nós Propomos! Sertã
 
Teoria heterotrófica e autotrófica dos primeiros seres vivos..pptx
Teoria heterotrófica e autotrófica dos primeiros seres vivos..pptxTeoria heterotrófica e autotrófica dos primeiros seres vivos..pptx
Teoria heterotrófica e autotrófica dos primeiros seres vivos..pptx
 
PROVA - ESTUDO CONTEMPORÂNEO E TRANSVERSAL: LEITURA DE IMAGENS, GRÁFICOS E MA...
PROVA - ESTUDO CONTEMPORÂNEO E TRANSVERSAL: LEITURA DE IMAGENS, GRÁFICOS E MA...PROVA - ESTUDO CONTEMPORÂNEO E TRANSVERSAL: LEITURA DE IMAGENS, GRÁFICOS E MA...
PROVA - ESTUDO CONTEMPORÂNEO E TRANSVERSAL: LEITURA DE IMAGENS, GRÁFICOS E MA...
 

Probabilidades e eventos

  • 1. MÓDULO I – PARTE 6 MATEMÁTICA Projeto Vestibular Noções de Prof. Bruno Vianna Probabilidade Noções de Probabilidade Dividindo todos os membros da desigualdade por n(U), vem: Experimento determinístico 0 n( A ) n(U) ≤ ≤ ∴ 0 ≤ P( A ) ≤ 1 n(U) n(U) n(U) Experimentos que ao serem realizados repetidas vezes em condições consideradas idênticas, apresentam resultados essencialmente idênticos são Probabilidade de Não Ocorrer Um Evento denominados experimentos determinísticos. Sendo A evento complementar do evento A do espaço amostral U, temos: Experimento aleatório Experimentos que ao serem realizados P( A ) + P( A ) = 1 repetidas vezes em condições consideradas idênticas, apresentam resultados diferentes, não sendo possível portanto a previsão lógica dos resultados, são Exercícios Resolvidos denominados experimentos aleatórios (ou casuais). 01) Uma urna contém 15 bolas numeradas de 1 a 15. Uma bola é extraída ao acaso da urna. Qual a Espaço Amostral probabilidade de ser sorteada uma bola com número maior ou igual a 11? É o conjunto de todos os resultados possíveis de um experimento aleatório. Indicaremos o espaço Espaço amostral U = {1,2,3,...,13,14,15} amostral por U. Evento requerido A = {11,12,13,14,15} (Nºs maiores ou iguais a 11) Evento n(A) = 5 É qualquer subconjunto do espaço amostral. n(U) = 15 O conjunto Ø é chamado evento impossível. O conjunto espaço amostral U é também um n( A) 5 1 evento, chamado de evento certo. p( A) = = = ≅ 33,3% Os subconjuntos unitários de U são chamados n(U ) 15 3 eventos elementares ou eventos simples. É certo que também podemos simplificar a idéia de probabilidade quando as situações estudadas são Espaço Amostral Eqüiprovável de fácil compreensão: n º de casos favoráveis 5 1 O espaço amostral de um experimento aleatório p= = = ≅ 33,3% é chamado eqüiprovável se todos os seus eventos n º total de casos 15 3 elementares têm a mesma chance de ocorrer 02) Um dado é lançado e observa-se o número da face Probabilidade voltada para cima. Qual a probabilidade desse número ser: Seja U um espaço amostral eqüiprovável e A 2 1 um de seus eventos. Denomina-se probabilidade do a) menor que 3? p= = ≅ 33,3% evento A o número P(A) tal que: 6 3 n( A ) 4 2 P( A ) = b) maior ou igual a 3? p = = ≅ 66,6% n(U) 6 3 onde: n(A) = número de elementos do evento A. 1 2 n(U) = número de elementos do espaço Observe que P(A) + P(B) = + =1 amostral U. 3 3 Ou seja, como P( A ) + P( A ) = 1 , temos que P( B) = P( A) Como A é subconjunto de U, decorre que: 0 ≤ n(A) ≤ n(U) 2011 1
  • 2. MÓDULO I – PARTE 6 MATEMÁTICA Projeto Vestibular Noções de Prof. Bruno Vianna Probabilidade Probabilidade da união de eventos 12 8 4 16 p( A ∪ B) = + − = = 0,64 = 64% 25 25 25 25 Se A e B são eventos quaisquer de um experimento aleatório do mesmo espaço amostral U, b) Qual a probabilidade do nº da bola sorteada ser então: múltiplo de 5 ou de 7? n( A U B) = n( A ) + n(B) − n( A I B) 5 Múltiplos de 5 > p ( A) = 25 Dividindo ambos os membros dessa igualdade 3 por n(U), temos: Múltiplos de 7 > p ( B ) = 25 n( A U B) n( A ) n(B) n( A I B) = + − n(U) n(U) n(U) n(U) Múltiplos de 5 e 7 > p( A ∩ B) = Ø Onde concluímos que: 5 3 8 p( A ∪ B) = + = = 0,32 = 32% P( A U B) = P( A ) + P(B) − P( A I B) 25 25 25 Pode ocorrer que os eventos A e B do Probabilidade Condicional espaço amostral U não tenham elementos comuns. Nesse caso, são chamados de eventos mutuamente Denomina-se probabilidade de B exclusivos ( ou eventos disjuntos ). Quando isso condicionada a A a probabilidade de ocorrência do ocorre, temos: evento B, sabendo que vai ocorrer ou que já ocorreu o evento A. Representaremos esse caso por P( B | A ) A IB = { } ⇒ P(A I B) = 0 (lê-se probabilidade de B dado A ). Logo, se A e B são eventos mutuamente exclusivos, temos: U B A P( A U B) = P( A ) + P(B) Resumindo: p( A ∪ B ) = p ( A) + p( B) ⇔ A ∩ B = Ø A∩ ou B p( A ∪ B ) = p ( A) + p( B) − p( A ∩ B ) ⇔ A ∩ B ≠ Ø Observe que, sabendo que o evento A ocorreu, então os casos favoráveis à ocorrência do evento B Exercício Resolvido: estão em A ∩ B. Temos então: 03) Uma urna contém 25 bolas numeradas de 1 a 25. n( A I B) Uma bola é extraída ao acaso. P(B | A ) = n( A ) a) Qual a probabilidade de o nº da bola sorteada ser múltiplo de 2 ou de 3? Dividindo numerador e denominador do segundo membro da igualdade por n(U), temos: 12 Múltiplos de 2 > p ( A) = n( A I B) 25 n(U) P( A I B ) P(B | A ) = ==> P(B | A ) = 8 n( A ) P( A ) Múltiplos de 3> p( B ) = 25 n(U) 4 Logo: Múltiplos de 2 e 3 > p( A ∩ B) = 25 P( A I B ) = P( A ) ⋅ P(B | A ) 2011 2
  • 3. MÓDULO I – PARTE 6 MATEMÁTICA Projeto Vestibular Noções de Prof. Bruno Vianna Probabilidade Então, para a ocorrência ao mesmo tempo de 02) (UNI-RIO) – O dispositivo que aciona a abertura do dois eventos, temos que a probabilidade de ocorrer A e cofre de uma joalheria apresenta um teclado com nove B é igual à probabilidade de ocorrer A multiplicada pela teclas, sendo cinco algarismos (0,1,2,3,4) e quatro letras probabilidade condicional de B dado A. (x,y,z,w). O segredo do cofre é uma seqüência de três algarismos seguidos de duas letras. Qual a Os eventos A e B são chamados eventos probabilidade de uma pessoa, numa única tentativa, ao independentes ou seja, a ocorrência de um evento não acaso, abrir o cofre ? depende da ocorrência do outro, quando vale a igualdade: (A) 1 / 7 200 (B) 1 / 1 500 P( A I B) = P( A ) ⋅ P(B) (C) 1 / 2 000 (D) 1 / 720 (E) 1 / 200 Exercícios Resolvidos 03) (UNIRIO-2000) Numa urna existem bolas de 04) Uma urna contém exatamente sete bolas: quatro plástico, todas do mesmo tamanho e peso, numeradas azuis e três vermelhas. Retira-se ao acaso uma bola da de 2 a 21, inclusive e sem repetição. A probabilidade de urna, registra-se sua cor e repõe-se a bola da urna. A se sortear um número primo ao pegarmos uma única seguir, retira-se novamente uma bola da urna e registra- bola, aleatoriamente, é de: se sua cor. Calcular a probabilidade de: (A) 45% (B) 40% (C) 35% a) sair uma bola azul e outra vermelha. (D) 30% (E) 25% 04) (UERJ-02) Em uma experiência de fecundação in vitro, 4 óvulos humanos, quando incubados com 4 suspensões de espermatozóides, todos igualmente Queremos que a primeira bola retirada seja azul e a viáveis, geraram 4 embriões, de acordo com a tabela segunda seja vermelha. A probabilidade de a primeira abaixo. bola ser azul é 4 , e a probabilidade de a segunda bola 7 sair vermelha é 3 . Assim, a probabilidade de obtermos 7 a sequência: A e V é P = 4 ⋅ 3 = 12 7 7 49 b) saírem duas bolas de cores diferentes. Observe os gráficos: Temos duas sequências possíveis, com as respectivas probabilidades: A e V → P1 = 4 ⋅ 3 = 12 OU V e A → P2 = 3 ⋅ 4 = 12 7 7 49 7 7 49 Assim a probabilidade total é: P = P1 + P2 = 12 + 12 = 24 Considerando a experiência descrita, o gráfico que 49 49 49 indica as probabilidades de os 4 embriões serem do sexo masculino é o de número: Exercícios Propostos (A) 1 (B) 2 (C) 3 (D) 4 01) Estudando Genética, os alunos da E.A. Corcovado construíram o quadro ao lado, em que os quatro 05) (UERJ-06-2ºex) Com o intuito de separar o lixo para eventos são prováveis. Qual a probabilidade de que fins de reciclagem, uma instituição colocou em suas ocorra o evento aa (em que o filho de um casal híbrido dependências cinco lixeiras de diferentes cores, de de olhos castanhos teria olhos azuis) ? acordo com o tipo de resíduo a que se destinam: vidro, Masc Fem A a plástico, metal, papel e lixo orgânico. A AA (castanho) Aa (castanho) A Aa (castanho) aa (azul) (A) 50% (B) 25% (C) 75% (D) 10% (E) 20% 2011 3
  • 4. MÓDULO I – PARTE 6 MATEMÁTICA Projeto Vestibular Noções de Prof. Bruno Vianna Probabilidade Sem olhar para as lixeiras, João joga em uma delas 11) (UERJ) uma embalagem plástica e, ao mesmo tempo, em outra, uma garrafa de vidro. A probabilidade de que ele tenha usado corretamente pelo menos uma lixeira é igual a: (A) 25% (B) 30% (C) 35% (D) 40% 06) (OBMEP-05) Brasil e Argentina participam de um campeonato internacional de futebol no qual competem oito seleções. Na primeira rodada serão realizadas quatro partidas, nas quais os adversários são escolhidos por sorteio. Qual é a probabilidade de Brasil e Argentina se enfrentarem na primeira rodada? Protéticos e dentistas dizem que a procura por (A) 1/8 (B) 1/7 (C) 1/6 dentes postiços não aumentou. Até declinou um (D) 1/5 (E) 1/4 pouquinho. No Brasil, segundo a Associação Brasileira de Odontologia (ABO), há 1,4 milhão de pessoas sem 07) (PM-05-1) Pedro brinca com um dado com seus nenhum dente na boca e 80% delas já usam dentadura. amigos. Ele não gosta do número 3. Se Pedro lançar o Assunto encerrado. dado duas vezes, a probabilidade de que o número 3 (Adaptado de Veja, outubro/97) não apareça em nenhum dos lançamentos é de, aproximadamente: Considere que a população seja de 160 milhões de habitantes. (A) 40% (B) 50% (C) 60% (D) 70% Escolhendo ao acaso um desses habitantes, a probabilidade de que ele não possua nenhum dente na 08) (PM-04-2) Em certo quartel, a probabilidade de um boca e use dentadura, de acordo com a ABO, é de: soldado ser torcedor do Flamengo é 0,60 e de gostar de praticar exercício de tiro é 0,70. As probabilidades (A) 0,28%; (B) 0,56%; (C) 0,70%; (D) 0,80%. mínima e máxima de um soldado deste quartel ser torcedor do Flamengo e, simultaneamente, gostar de praticar exercícios de tiro, são, respectivamente: 12) Numa urna contendo 5 bolas brancas e 10 bolas pretas, cada vez que se retira uma delas procede-se da (A) 10% e 60% (B) 20% e 60% seguinte maneira: (C) 30% e 60% (D) 40% e 60% − Se a bola for branca: não se repõe esta bola, porém acrescenta-se 6 outras bolas pretas; 09) (PM-04-2) Um comandante deseja premiar três dos − Se a bola for preta: repõe-se esta bola sete soldados mais qualificados de seu quartel, juntamente com outras 5 bolas brancas. adotando o critério de sorteio. Todos os soldados qualificados têm nomes diferentes e João e Pedro estão A probabilidade da SEGUNDA bola retirada desta urna ser entre eles. A probabilidade de João e Pedro serem dois branca, é: dos nomes sorteados é de: (A) 20% (B) 25% (C) 33,333...% (D) 40% (E) 50% (A) 1/7 (B) 2/7 (C) 3/7 (D) 4/7 13) Uma urna A contém x bolas vermelhas e y bolas 10) (UERJ-99) brancas. Uma urna B contém z bolas vermelhas e w bolas brancas. Uma bola é retirada da urna A e colocada na urna B e, então, uma bola é retirada da urna B. A probabilidade desse última bola ser vermelha é: z +1 x+z (O Dia, 25/08/98) (A) (B) z + 1+ w x+y+z+w Suponha haver uma probabilidade de 20% para uma caixa de Microvlar ser falsificada. Em duas caixas, a 1  x + xz + zy  1  xy + xz + zy  (C)   (D) x + y  z + w +1    probabilidade de pelo menos uma delas ser falsa é:   x + y  z + w +1  (A) 4 % (B) 16 % (C) 20 % (D) 36 % 2011 4
  • 5. MÓDULO I – PARTE 6 MATEMÁTICA Projeto Vestibular Noções de Prof. Bruno Vianna Probabilidade 14) (Enem-2001) Uma empresa de alimentos imprimiu 16) (PUC-RIO-2010) Quatro moedas são lançadas em suas embalagens um cartão de apostas do seguinte simultaneamente. Qual é a probabilidade de ocorrer tipo: coroa em uma só moeda? 1 2 1 (A) (B) (C) 8 9 4 1 3 (D) (E) 3 8 17) (PUC-RIO-2011) Jogamos três dados comuns simultaneamente. Qual a probabilidade de que os três números sorteados sejam distintos? 1 1 5 (A) (B) (C) 2 36 9 17 5 (D) (E) 36 17 18) (Enem-2001) Um município de 628 km² é atendido por duas emissoras de rádio cujas antenas A e B alcançam um raio de 10km do município, conforme mostra a figura: Para orçar um contrato publicitário, uma agência precisa avaliar a probabilidade que um morador tem de, Cada cartão de apostas possui 7 figuras de bolas de circulando livremente pelo município, encontrar-se na futebol e 8 sinais de “X” distribuídos entre os 15 área de alcance de pelo menos uma das emissoras. espaços possíveis, de tal forma que a probabilidade de um cliente ganhar o prêmio nunca seja igual a zero. Essa probabilidade é de, aproximadamente, Em determinado cartão existem duas bolas na linha 4 e duas bolas na linha 5. Com esse cartão, a probabilidade (A) 20%. (B) 25%. (C) 30%. de o cliente ganhar o prêmio é (D) 35%. (E) 40%. (A)1/27. (B) 1/36. (C) 1/54. 19) (PUC-RIO-2011) Considere uma urna contendo (D)1/72. (E) 1/108. vinte bolas numeradas de 1 a 20. Retiram-se três bolas simultaneamente e de maneira aleatória de dentro desta 15) A figura abaixo representa um alvo de dardos, urna. composto de três círculos concêntricos de raios r, 2r e 3r. Sabendo que um competidor acertou o alvo, qual é a) Qual a probabilidade de que a soma seja igual a 6? a probabilidade dele ter acertado a parte clara do alvo? b) Qual a probabilidade de que a soma seja igual a 8? c) Qual a probabilidade de que a soma seja igual a 15? 20) (PUC-RIO-2011) Considere uma urna contendo 5 bolas pretas e 5 bolas brancas. Retiram-se simultaneamente e de maneira aleatória 3 bolas de dentro desta urna. a) Qual a probabilidade de que todas as bolas retiradas sejam brancas? (A) 1/3 (B) 1/2 (C) 1/4 b) Qual a probabilidade de que, entre as bolas retiradas, (D) 1/9 (E) 4/9 duas bolas sejam brancas e uma bola seja preta? 2011 5
  • 6. MÓDULO I – PARTE 6 MATEMÁTICA Projeto Vestibular Noções de Prof. Bruno Vianna Probabilidade 21) (ENEM-2010) O diretor de um colégio leu numa Em 2050, a probabilidade de se escolher, aleatoriamente, revista que os pés das mulheres estavam aumentando. uma pessoa com 60 anos ou mais de idade, na população Há alguns anos, a média do tamanho dos calçados das dos países desenvolvidos, será um número mais próximo mulheres era de 35,5 e, hoje, é de 37,0. Embora não de fosse uma informação científica, ele ficou curioso e fez 1 7 8 uma pesquisa com as funcionárias do seu colégio, (A) (B) (C) 2 20 25 obtendo o quadro a seguir: 1 3 (D) (E) Escolhendo uma funcionária ao acaso e sabendo que 5 25 ela tem calçado maior que 36,0, a probabilidade de ela calçar 38,0 é: 23) (ENEM-09) O controle de qualidade de uma empresa fabricante de telefones celulares aponta que a probabilidade de um aparelho de determinado modelo apresentar defeito de fabricação é de 0,2%. Se uma loja acaba de vender 4 aparelhos desse modelo para um cliente, qual é a probabilidade de esse cliente sair da loja com exatamente dois aparelhos defeituosos? 4 2 (A) 2 × (0,2%) . (B) 4 × (0,2%) . 2 2 1 1 2 (C) 6 × (0,2%) × (99,8%) . (D) 4 × (0,2%). (A) (B) (C) 3 5 5 (E) 6 × (0,2%) × (99,8%). 5 5 (D) (E) 7 14 24) (ENEM-09) A população brasileira sabe, pelo menos intuitivamente, que a probabilidade de acertar as seis 22) (ENEM-09) A população mundial está ficando mais dezenas da mega sena não é zero, mas é quase. velha, os índices de natalidade diminuíram e a expectativa Mesmo assim, milhões de pessoas são atraídas por de vida aumentou. No gráfico seguinte, são apresentados essa loteria, especialmente quando o prêmio se dados obtidos por pesquisa realizada pela Organização acumula em valores altos. Até junho de 2009, cada das Nações Unidas (ONU), a respeito da quantidade de aposta de seis dezenas, pertencentes ao conjunto {01, pessoas com 60 anos ou mais em todo o mundo. Os 02, 03, ..., 59, 60}, custava R$ 1,50. números da coluna da direita representam as faixas Disponível em: www.caixa.gov.br. Acesso em: 7 jul. 2009. percentuais. Por exemplo, em 1950 havia 95 milhões de Considere que uma pessoa decida apostar exatamente pessoas com 60 anos ou mais nos países desenvolvidos, R$ 126,00 e que esteja mais interessada em acertar número entre 10% e 15% da população total nos países desenvolvidos. apenas cinco das seis dezenas da mega sena, justamente pela dificuldade desta última. Nesse caso, é melhor que essa pessoa faça 84 apostas de seis dezenas diferentes, que não tenham cinco números em comum, do que uma única aposta com nove dezenas, porque a probabilidade de acertar a quina no segundo caso em relação ao primeiro é, aproximadamente, 1 1 (A) 1 vez menor (B) 2 vezes menor 2 2 (C) 4 vezes menor (D) 9 vezes menor (E) 14 vezes menor 25) (UERJ-2011 -1º ex qualif) Uma fábrica produz sucos com os seguintes sabores: uva, pêssego e laranja. Considere uma caixa com 12 garrafas desses sucos, sendo 4 garrafas de cada sabor. Retirando-se, ao acaso, 2 garrafas dessa caixa, a probabilidade de que ambas contenham suco com o mesmo sabor equivale a: (A) 9,1% (B) 18,2% Disponível em: www.economist.com. Acesso em: 9 jul. 2009 (adaptado). (C) 27,3% (D) 36,4% 2011 6
  • 7. MÓDULO I – PARTE 6 MATEMÁTICA Projeto Vestibular Noções de Prof. Bruno Vianna Probabilidade 26) (UERJ 2011- 2ºex qualif) Uma máquina contém 31) (UFF-2010-2ºF) Dois dados cúbicos não viciados, pequenas bolas de borracha de 10 cores diferentes, cujas faces estão numeradas de 1 a 6, são jogados sendo 10 bolas de cada cor. Ao inserir uma moeda na aleatoriamente e simultaneamente sobre uma mesa máquina, uma bola é expelida ao acaso. plana. Se a soma dos valores sorteados(*) for um Observe a ilustração: número par, Paulo ganha a partida. Se a soma for um número ímpar, Lúcia ganha. Ao perder a primeira partida, Lúcia diz que não irá mais jogar porque a regra favorece Paulo. Seu argumento é o seguinte: dentre os onze valores possíveis para a soma (os inteiros de 2 a 12), há seis números pares e apenas cinco números ímpares. Logo, Paulo tem maior probabilidade de Para garantir a retirada de 4 bolas de uma mesma cor, o ganhar. menor número de moedas a serem inseridas na máquina corresponde a: a) Calcule a probabilidade de Lúcia ganhar uma partida. Justifique sua resposta. (A) 5 (B) 13 (C) 31 (D) 40 b) Use o item a para verificar se o argumento de Lúcia 27) (UERJ 2011- 2ºex qualif) Inserindo-se 3 moedas, está correto. uma de cada vez, a probabilidade de que a máquina (*) Valor sorteado é o número escrito na face do cubo libere 3 bolas, sendo apenas duas delas brancas, é oposta à face que está apoiada na mesa. aproximadamente de: 32) (PUC-2010 – 2ª f) (A) 0,008 (B) 0,025 Considere o lançamento de três dados comuns. (C) 0,040 (D) 0,072 a) Qual é a probabilidade de que a soma dos valores 28) (UFRJ-2004-PE) Manoel e Joaquim resolveram sorteados seja igual a 5? disputar o seguinte jogo: uma bola será retirada ao acaso de uma urna que contém 999 bolas idênticas, b) Qual é a probabilidade de que os três números numeradas de 1 a 999. Se o número sorteado for par, sorteados sejam diferentes? ganha Manoel; se for ímpar Joaquim ganha. Isto foi resolvido após muita discussão, pois ambos queriam as 33) (UERJ-2011-2ª FASE) Para a realização de uma pares. partida de futebol são necessários três árbitros: um juiz Se todas as bolas tem a mesma principal, que apita o jogo, e seus dois auxiliares, que probabilidade de serem retiradas, identifique quem ficam nas laterais. Suponha que esse trio de arbitragem tem mais chances de ganhar o jogo. Justifique sua seja escolhido aleatoriamente em um grupo composto resposta. de somente dez árbitros, sendo X um deles. Após essa escolha, um segundo sorteio aleatório é feito entre os 29) (UFRJ-98-PE) Duzentas bolas pretas e duzentas três para determinar qual deles será o juiz principal. bolas brancas são distribuídas em duas urnas, de modo que cada uma delas contenha cem bolas pretas e cem Calcule a probabilidade de X ser o juiz principal. bolas brancas. Uma pessoa retira ao acaso uma bola de cada urna. 34) (UERJ-2007-ESP) João recorta um círculo de papel com 10 cm de raio. Em seguida, dobra esse recorte ao Determine a probabilidade de que as duas meio várias vezes, conforme ilustrado abaixo. bolas retiradas sejam de cores distintas. 30) (UFRJ-2009) João criou uma senha de 4 algarismos para o segredo de seu cofre. Mais tarde, quando foi abrir o cofre, João percebeu que não lembrava mais qual era a senha, mas sabia que os algarismos eram 1, 3, 8 e 9. Ele, então, resolveu escrever todos os números possíveis formados pelos 4 algarismos e, em seguida, tentar abrir o cofre sorteando ao acaso, um a um, os números de sua lista, sem repetir números já testados. a) Determine quantos números João escreveu. b) Calcule a probabilidade de que ele abra o cofre na 12ª tentativa. 2011 7
  • 8. MÓDULO I – PARTE 6 MATEMÁTICA Projeto Vestibular Noções de Prof. Bruno Vianna Probabilidade Depois de fazer diversas dobras, abre o papel e coloca o número 1 nas duas extremidades da primeira dobra. Sucessivamente, no meio de cada um dos arcos formados pelas dobras anteriores, João escreve a soma dos números que estão nas extremidades de cada arco. As figuras a seguir ilustram as quatro etapas iniciais desse processo. Uma criança rasgou algumas cartas desse baralho, e as n cartas restantes, não rasgadas, foram guardadas em uma caixa. A tabela abaixo apresenta as probabilidades de retirar- se dessa caixa, ao acaso, as seguintes cartas: Calcule o valor de n. 36) (UERJ-2010-ESP) Uma criança guarda moedas de R$ 1,00 e de R$ 0,50 em duas caixas, uma verde e outra amarela. Na caixa amarela, há, exatamente, A figura correspondente à etapa 3 foi colada em uma 12 moedas de R$ 1,00 e 15 moedas de R$ 0,50. roleta, que após ser girada pode parar, ao acaso, em apenas oito posições distintas. Uma seta indica o Admita que, após a transferência de n moedas de R$ número correspondente a cada posição, como ilustra a 1,00 da caixa verde para a amarela, a probabilidade figura abaixo. de se retirar ao acaso uma moeda de R$ 1,00 da caixa amarela seja igual a 50%. Calcule o valor de n. 37) (UFRJ-2010) Um ponto P é aleatoriamente selecionado num retângulo S de dimensões 50 cm por 20 cm. Considere, a partir de S, as seguintes regiões: Região A – retângulo de dimensões 15 cm por 4 cm João girou a roleta duas vezes consecutivas e anotou com centro no centro de S e os números indicados pela seta após cada parada. Região B – círculo de raio 4 cm com centro no centro de Calcule a probabilidade de a soma desses números S. ser par. Suponha que a probabilidade de que o ponto P 35) (UERJ-2009-ESP) pertença a uma região contida em S seja proporcional à Os baralhos comuns são compostos de 52 cartas área da região. divididas em quatro naipes, denominados copas, Determine a probabilidade de que P pertença espadas, paus e ouros, com treze cartas distintas de simultaneamente às regiões A e B. cada um deles. Observe a figura que mostra um desses baralhos, no 38) (UFRJ-2011) Um ponto M é selecionado ao acaso qual as cartas representadas pelas letras A, J, Q e K no interior de um círculo C de raio 2 e centro O. Em são denominadas, respectivamente, ás, valete, dama e seguida, constrói-se um quadrado, também centrado em rei. O, que tem M como ponto médio de um de seus lados. Calcule a probabilidade de que o quadrado assim construído esteja inteiramente contido no círculo C. 2011 8
  • 9. MÓDULO I – PARTE 6 MATEMÁTICA Projeto Vestibular Noções de Prof. Bruno Vianna Probabilidade 39) (PUC-2010 – 2ª f) O diagrama abaixo mostra uma 40) (UNICAMP - 2002) Em Matemática, um número sala do jogo Os Labirintos da Simetria. Isaac, o herói do natural a é chamado palíndromo se seus algarismos, jogo, entra na sala por um portão no extremo esquerdo escritos em ordem inversa, produzem o mesmo número. da sala e precisa sair pelo portão que está no extremo Por exemplo, 8, 22 e 373 são palíndromos. Pergunta-se: direito da sala e que inicialmente está fechado. a) Quantos números naturais palíndromos existem entre 1 e 9.999? b) Escolhendo-se ao acaso um número natural entre 1 e 9.999, qual é a probabilidade de que esse número seja No corredor entre os dois portões há sete cristais, cada palíndromo? Tal probabilidade é maior ou menor que um com uma cor do arco íris: Vermelho, Laranja, 2%? Justifique sua resposta. Amarelo, Verde, Azul, Índigo e Violeta. A cada partida as posições dos cristais são sorteadas, com igual GABARITOS probabilidade para cada uma das ordens possíveis. Para que o portão de saída se abra, Isaac precisa tocar 01) B 02) C 03) B 04) A os sete cristais exatamente na ordem acima. Na sala há uma corrente de ar da esquerda para a direita. Assim, 05) C 06) B 07) D 08) C Isaac pode mover-se facilmente da esquerda para a direita, mas para mover-se da direita para a esquerda 09) A 10) D 11) C 12) D ele precisa acionar as suas Hélices Mágicas. Cada vez que ele aciona as Hélices ele gasta uma carga. Para 13) C 14) C 15) A 16) C tocar um cristal, Isaac deve desligar as Hélices e se depois de tocar um cristal ele precisar se mover 18) B 19) a) 1/1140 b) 1/570 c) 1/95 novamente para a esquerda ele precisará gastar outra carga. 20) a) 1/12 b) 5/12 21) D 22) C Assim, por exemplo, se num jogo a posição dos cristais 23) C 24) C 25) C 26) C for: 27) B Amarelo - Laranja - Índigo - Verde - Violeta - Vermelho – Azul 28) Joaquim tem mais chances de ganhar o jogo, já que há 500 bolas com números ímpares e 499 bolas com então Isaac chegará gratuitamente ao cristal Vermelho, números pares. gastará uma carga para voltar até Laranja e uma segunda para voltar até Amarelo. Depois disso ele se 29) 50% 30) a) 24 b) 1/24 moverá gratuitamente até Verde e daí até Azul. Isaac gastará uma terceira carga para voltar até Índigo e 31) a) 50% b) 50% 32) a) 1/36 b) 5/9 depois se moverá gratuitamente até Violeta e de lá para o portão de saída, finalmente aberto. Neste exemplo, 33) 1/10 34) 5/8 35) n = 40 para passar pela sala, Isaac gastou três cargas. 16π + 24 3 Considerando agora uma sala com cristais em posições 36) n = 3 37) P = 38) ½ sorteadas, responda: 3000 39) a) 1/7! = 1/5040 b) 6/7! = 1/840 c) 120/7! = 1/42 a) Qual a probabilidade de que Isaac possa passar pela sala sem gastar nenhuma carga? 40) a) 196 b) 1,96 b) Qual a probabilidade de que Isaac passe pela sala gastando uma carga para ir de Vermelho até Laranja e depois não precise gastar mais nenhuma outra carga? c) Qual a probabilidade de que Isaac precise gastar exatamente uma carga para passar pela sala? 2011 9
  • 10. MÓDULO I – PARTE 6 MATEMÁTICA Projeto Vestibular Noções de Prof. Bruno Vianna Probabilidade Algumas resoluções: Questão 30) Questão 8) a) São 4 algarismos distintos. Tem-se que 4! = 24. João escreveu 24 números. Fla Tiro b)Solução da Banca: Olhando-se uma lista qualquer dos 24 números 60 - x x 70 - x possíveis, observe que a probabilidade da senha correta estar na n-ésima posição não depende de n. Deste modo a probabilidade de João acertar na 12ª tentativa é 60 – x + x + 70 –x = 100 >> x = 30% igual à probabilidade de João acertar na primeira, que é 1/24 Como x ≥ 0 >> x ≤ 60% Solução mais simples: Para que João acerte apenas na 12ª tentativa, Questão 9) obrigatoriamente ele deve errar as onze tentativas anteriores e acertar a 12ª, logo: C 5,1 5 1 23 22 21 20 19 18 17 16 15 14 13 1 Resol: = = P= ⋅ ⋅ ⋅ ⋅ ⋅ ⋅ ⋅ ⋅ ⋅ ⋅ ⋅ C 7 ,3 35 7 24 23 22 21 20 19 18 17 16 15 14 13 As cinco são : João Pedro __ ____ ____ ____ ____ 1 P= Questão 19) 24 Há C20,3 = 1140 maneiras de se retirarem 3 bolas da Questão 31) urna. a) O espaço amostral desse experimento é o conjunto a) Soma igual a 6: 1 + 2 + 3 (somente um maneira). A, com 36 elementos: Logo P(a) = 1/1140. A = { (1, 1), (1, 2), (1, 3), (1, 4), (1, 5), (1, 6),(2, 1), (2, 2), b) Soma igual a 8: 1 + 2 + 5 e 1 + 3 + 4 (duas (2, 3), (2, 4), (2, 5), (2, 6),(3, 1), (3, 2), (3, 3), (3, 4), (3, maneiras). Logo P (b) = 2/1140 = 1/570. 5), (3, 6),(4, 1), (4, 2), (4, 3), (4, 4), (4, 5), (4, 6),(5, 1), (5, 2), (5, 3), (5, 4), (5, 5), (5, 6),(6, 1), (6, 2), (6, 3), (6, c) Soma igual a 15: 4), (6, 5), (6, 6) }. 1 + 2 + 12, 1 + 3 + 11, 1 + 4 + 10, 1 + 5 + 9, 1 + 6 + 8, 2 + 3 + 10, 2 + 4 + 9, O evento “a soma dos valores sorteados é um número 2 + 5 + 8, 2 + 6 + 7, 3 + 4 + 8, 3 + 5 + 7, 4 + 5 + 6 (doze ímpar” é o conjunto E, com 18 elementos: maneiras). Logo P (c) = 12/1140 = 1/95. E = { (1, 2), (1, 4), (1, 6),(2, 1), (2, 3), (2, 5),(3, 2), (3, 4), Questão 20) (3, 6),(4, 1), (4, 3), (4, 5),(5, 2), (5, 4), (5, 6),(6, 1), (6, 3), Há C10,3 =120 maneiras de se retirarem 3 bolas da (6, 5) }. urna. Logo, a probabilidade de Lúcia ganhar é igual a 18/36 = C 10 1 a) Tirar três bolas brancas: P (a ) = 5,3 = = 1/2 = 50%. C10,3 120 12 b) O cálculo feito no item (a) mostra que Paulo e Lúcia b) Tirar duas brancas e uma preta: têm a mesma probabilidade de ganhar uma partida. C ⋅C 50 5 Questão 32) P(b) = 5, 2 5,1 = = Temos no lançamento de três dados 63 possibilidades. C10,3 120 12 a) O evento ter soma 5, tem casos : (1,2,2), (2,2,1) , Questão 29) Qualquer que seja a cor da bola retirada (1,2,1),(1,1,3),(1,3,1) e (3,1,1) então, na primeira urna, a chance de se retirar uma bola de cor P= 6/216 = 1/36 diferente da segunda urna é de 100/200. b) O evento ter todos os números diferentes, vale 6 × 5 Logo: P = ½ = 50% × 4. Logo, P = (6.5.4)/216 = 5/9 2011 10
  • 11. MÓDULO I – PARTE 6 MATEMÁTICA Projeto Vestibular Noções de Prof. Bruno Vianna Probabilidade Questão 33) A = {x; a2 ; a3 ; a4 ; ...; a10} 9 ⋅8 C9, 2 = 2 ⋅1 = 36 3 1º sorteio: P( x) = = C10,3 10 ⋅ 9 ⋅ 8 120 10 3 ⋅ 2 ⋅1 1 3 1 Observando-se o triângulo retângulo OLN, tem-se que o P( xJUÍZ ) = ⋅ = ângulo LÔN mede 60º. Assim, a medida da área do 3 10 10 8π 2 setor circular OMN é cm e a área do triângulo OLN Questão 34) 3 1 é 2 3 cm . 2 Probabilidade de ocorrer par e par ⇒ P1 = 16 Portanto, a medida da área da região C Probabilidade de ocorrer ímpar e ímpar ⇒ P2 = 9  8π  16 é − 2 3  cm2 .  3  Probabilidade de ocorrer soma par ⇒ Logo, a medida da área de A∩ B é: 10 5 P1 + P2 = =   8π  2 16 8 16π − 4 3 − 2 3  cm    2 Questão 35) Como a medida da área de S é 1000 cm , tem-se que a 16π + 24 3 Número de cartas guardadas na caixa: n probabilidade solicitada é P = . 3000 Probabilidade de retirada de: - um rei → P(R) = 0,075 - uma carta de copas → P(C) = 0,25 Questão 38) - um carta de copas ou um rei → P(C ∩ R) = 0,3 A diagonal do quadrado inscrito é igual ao diâmetro do - o rei de copas → P(R ∩ C) = P(R) + P(C) – P(R ∪ C) círculo C, ou seja, d = 4. A medida L do lado deste 2 quadrado é, por Pitágoras, 2L = 16 , ou seja, L = 2 2 . 1 P(R ∩ C) = 0,075 + 0,25 − 0,3 = 0,025 = n 1 1 = ⇒ n = 40 n 40 Questão 36) 1 12 + n = Para que o quadrado esteja inteiramente contido em C, 2 12 + n + 15 a distância de M ao centro de C deve ser menor do que → 12 + n + 15 = 2 (12 + n) → L → n + 27 = 24 + 2n → ou igual a . Ou seja, M pertence a um círculo CM de → 27 – 24 = 2n – n → 2 → n=3 L raio e mesmo centro C. 2 Questão 37) Por hipótese, a probabilidade de que o ponto P pertença Então a probabilidade pedida é: a uma região F, contida em S, é dada pela razão entre a medida da área de F e a medida da área de S. área (CM ) 2π 1 Assim, a probabilidade de que o ponto P pertença a p= = = área ( A ∩ B) área (C ) 4π 2 ambas as regiões é dada por: área ( S ) Seja C a região sombreada na figura abaixo. Então,a área (A∩B) = 16π – 4 × área (C). 2011 11
  • 12. MÓDULO I – PARTE 6 MATEMÁTICA Projeto Vestibular Noções de Prof. Bruno Vianna Probabilidade Questão 39) a) Isto só ocorrerá se os cristais estiverem na ordem: Vermelho - Laranja - Amarelo - Verde - Azul - Índigo - Violeta A probabilidade de isso ocorrer é 1/7! = 1/5040. b) Isto ocorrerá se as cores: Laranja - Amarelo - Verde - Azul - Índigo – Violeta aparecerem nesta ordem da esquerda para a direita, com Vermelho em qualquer posição exceto na primeira. Há, assim, 6 configurações possíveis e a probabilidade pedida é 6/7! = 1/840. c) Para formar uma configuração deste tipo, devemos 7 primeiro selecionar um conjunto de posições (há 2 = 128 maneiras de fazer isso). Primeiro preenchemos as posições do conjunto da esquerda para a direita com as cores na ordem em que Isaac deve tocá-las e depois preenchemos as posições no complemento do conjunto. Isto só *não* funcionará se as posições do conjunto estiverem todas à esquerda das posições do complemento (pois neste caso Isaac não gastaria nenhuma carga), ou seja, para os 8 conjuntos {}, {1}, {1,2}, {1,2,3}, ..., {1,2,3,4,5,6,7}. Assim há 128 - 8 = 120 configurações possíveis, e a probabilidade pedida é 120/7! = 1/42. Questão 40) a) De 1 até 9.999, temos desde palíndromos de 1 algarismo até palíndromos de 4 algarismos. Assim, x x x x ou ou ou 9 + 9 ⋅ 1 + 9 ⋅ 10 ⋅ 1 + 9 ⋅ 10 ⋅ 1 ⋅ 1 = 198 Considerando que “entre 1 e 9.999” não devam ser incluídos os extremos, temos 196 palíndromos. Resposta: 196 b) “Entre 1 e 9.999” temos 9.997 números. Assim, a probabilidade pedida é: 196 P= ≈ 1,96 % 9.997 Nota: Se interpretássemos o “entre 1 e 9.999” com a possibilidade da inclusão dos extremos, teríamos: a) 198 palíndromos. 198 2 b) P = = ≈ 1,98 %. 9.999 101 2011 12